HESI HEALTH ASSESSMENT LATEST 2023 TEST BANK REAL EXAM 300 QUESTIONS AND CORRECT DETAILED ANSWERS WITH RATIONALES |AGRADE(BRAND NEW!!)

Hesi health assessment exam questions and answers
Hesi health assessment exam questions
Hesi health assessment exam pdf
hesi health assessment test bank 2023
hesi health assessment exam 2023
health assessment hesi 55 questions
hesi health assessment exam quizlet
health assessment hesi 55 questions quizlet

  1. In an interview, the nurse may find it necessary to take notes to aid his or her memory later. Which statement is true regarding note-taking?

A) Note-taking may impede the nurse’s observation of the patient’s nonverbal behaviors.
B) Note-taking allows the patient to continue at his or her own pace as the nurse records what is said.
C) Note-taking allows the nurse to shift attention away from the patient, resulting in an increased comfort level.
D) Note-taking allows the nurse to break eye contact with the patient, which may increase his or her level of comfort.
A) Note-taking may impede the nurse’s observation of the patient’s nonverbal behaviors.

Page: 31 Some use of history forms and note-taking may be unavoidable. But be aware that note-taking during the interview has disadvantages. It breaks eye contact too often, and it shifts attention away from the patient, which diminishes his or her sense of importance. It also may interrupt the patient’s narrative flow, and it impedes the observation of the patient’s nonverbal behavior.

  1. During an interview, the nurse states, “You mentioned shortness of breath. Tell me more about that.” Which verbal skill is used with this statement?

A) Reflection
B) Facilitation
C) Direct question
D) Open-ended question
D) Open-ended question

Page: 32 The open-ended question asks for narrative information. It states the topic to be discussed but only in general terms. The nurse should use it to begin the interview, to introduce a new section of questions, and whenever the person introduces a new topic.

  1. A nurse is taking complete health histories on all of the patients attending a wellness workshop. On the history form, one of the written questions asks, “You don’t smoke, drink, or take drugs, do you?” This question is an example of:

A) talking too much.
B) using confrontation.
C) using biased or leading questions.
D) using blunt language to deal with distasteful topics.
C) using biased or leading questions.

Page: 36 This is an example of using leading or biased questions. Asking, “You don’t smoke, do you?” implies that one answer is “better” than another. If the person wants to please someone, he or she is either forced to answer in a way corresponding to their implied values or is made to feel guilty when admitting the other answer.

  1. During an interview, a parent of a hospitalized child is sitting in an open position. As the interviewer begins to discuss his son’s treatment, however, he suddenly crosses his arms against his chest and crosses his legs. This would suggest that the parent is:

A) just changing positions.
B) more comfortable in this position.
C) tired and needs a break from the interview.
D) uncomfortable talking about his son’s treatment.
D) uncomfortable talking about his son’s treatment.

Page: 37 Note the person’s position. An open position with the extension of large muscle groups shows relaxation, physical comfort, and a willingness to share information. A closed position with the arms and legs crossed tends to look defensive and anxious. Note any change in posture. If a person in a relaxed position suddenly tenses, it suggests possible discomfort with the new topic.

  1. The nurse is interviewing a patient who has a hearing impairment. What techniques would be most beneficial in communicating with this patient?

A) Determine the communication method he prefers.
B) Avoid using facial and hand gestures because most hearing-impaired people find this degrading.
C) Request a sign language interpreter before meeting with him to help facilitate the communication.
D) Speak loudly and with exaggerated facial movement when talking with him because this helps with lip reading.
A) Determine the communication method he prefers.

Pages: 40-41 The nurse should ask the deaf person the preferred way to communicate—by signing, lip reading, or writing. If the person prefers lip reading, then the nurse should be sure to face him or her squarely and have good lighting on the nurse’s face. The nurse should not exaggerate lip movements because this distorts words. Similarly, shouting distorts the reception of a hearing aid the person may wear. The nurse should speak slowly and should supplement his or her voice with appropriate hand gestures or pantomime.

  1. The nurse is performing a health interview on a patient who has a language barrier, and no interpreter is available. Which is the best example of an appropriate question for the nurse to ask in this situation?

A) “Do you take medicine?”
B) “Do you sterilize the bottles?”
C) “Do you have nausea and vomiting?”
D) “You have been taking your medicine, haven’t you?”
A) “Do you take medicine?”

Page: 46 In a situation where there is a language barrier and no interpreter available, use simple words avoiding medical jargon. Avoid using contractions and pronouns. Use nouns repeatedly and discuss one topic at a time.

  1. A female patient does not speak English well, and the nurse needs to choose an interpreter. Which of the following would be the most appropriate choice?

A) A trained interpreter
B) A male family member
C) A female family member
D) A volunteer college student from the foreign language studies department
A) A trained interpreter

Page: 46 whenever possible, the nurse should use a trained interpreter, preferably one who knows medical terminology. In general, an older, more mature interpreter is preferred to a younger, less experienced one, and the same gender is preferred when possible.

  1. The nurse is conducting an interview. Which of these statements is true regarding open-ended questions? Select all that apply.

A) They elicit cold facts.
B) They allow for self-expression.
C) They build and enhance rapport.
D) They leave interactions neutral.
E) They call for short one- to two-word answers.
F) They are used when narrative information is needed.
B) They allow for self-expression.
C) They build and enhance rapport.
F) They are used when narrative information

Page: 32 Open-ended questions allow for self-expression, build rapport, and obtain narrative information. These features enhance communication during an interview. The other statements are appropriate for closed or direct questions.

  1. The nurse is conducting an interview in an outpatient clinic and is using a computer to record data. Which is the best use of the computer in this situation? Select all that apply.

A) Collect the patient’s data in a direct, face-to-face manner.
B) Enter all the data as the patient states it.
C) Ask the patient to wait as the nurse enters data.
D) Type the data into the computer after the narrative is fully explored.
E) Allow the patient to see the monitor during typing.
A) Collect the patient’s data in a direct, face-to-face manner.
D) Type the data into the computer after the narrative is fully explored.
E) Allow the patient to see the monitor during typing.

Page: 32 The use of a computer can become a barrier. The nurse should begin the interview as usual by greeting the patient, establishing rapport, and collecting the patient’s narrative story in a direct face-to-face manner. Only after the narrative is fully explored should the nurse type data into the computer. When typing, the nurse should position the monitor so that the patient can see it.

  1. During an assessment, the nurse notices that a patient is handling a small charm that is tied to a leather strip around his neck. Which action by the nurse is appropriate?

A) Ask the patient about the item and its significance.
B) Ask the patient to lock the item with other valuables in the hospital’s safe.
C) Tell the patient that a family member should take valuables home.
D) No action is necessary.
A) Ask the patient about the item and its significance.

Page: 21 The nurse should inquire about the amulet’s meaning. Amulets, such as charms, are often seen as an important means of protection from “evil spirits” by some cultures.

  1. In the majority culture of America, coughing, sweating, and diarrhea are symptoms of an illness. For some individuals of Mexican-American origin, however, these symptoms are a normal part of living. The nurse recognizes that this is true, probably because Mexican-Americans:

A) have less efficient immune systems and are often ill.
B) consider these symptoms a part of normal living, not symptoms of ill health.
C) come from Mexico and coughing is normal and healthy there.
D) are usually in a lower socioeconomic group and are more likely to be sick.
B) consider these symptoms a part of normal living, not symptoms of ill health.

Page: 27 The nurse needs to identify the meaning of health to the patient, remembering that concepts are derived, in part, from the way in which members of the cultural group define health.

  1. Among many Asians there is a belief in the yin/yang theory, rooted in the ancient Chinese philosophy of Tao. The nurse recognizes which statement that most accurately reflects “health” in an Asian with this belief?

A) A person is able to work and produce.
B) A person is happy, stable, and feels good.
C) All aspects of the person are in perfect balance.
D) A person is able to care for others and function socially.
C) All aspects of the person are in perfect balance.

Page: 21 Many Asians believe in the yin/yang theory, in which health is believed to exist when all aspects of the person are in perfect balance. The other statements do not describe this theory.

  1. An individual who takes the magicoreligious perspective of illness and disease is likely to believe that his or her illness was caused by:

A) germs and viruses.
B) supernatural forces.
C) eating imbalanced foods.
D) an imbalance within his or her spiritual nature.
B) supernatural forces.

Page: 21 The basic premise of the magicoreligious perspective is that the world is seen as an arena in which supernatural forces dominate. The fate of the world and those in it depends on the actions of supernatural forces for good or evil. The other answers do not reflect the magicoreligious perspective.

  1. If an American Indian has come to the clinic to seek help with regulating her diabetes, the nurse can expect that she:

A) will comply with the treatment prescribed.
B) has obviously given up her beliefs in naturalistic causes of disease.
C) may also be seeking the assistance of a shaman or medicine man.
D) will need extra help in dealing with her illness and may be experiencing a crisis of faith.
C) may also be seeking the assistance of a shaman or medicine man.

Page: 23 When self-treatment is unsuccessful, the individual may turn to the lay or folk healing systems, to spiritual or religious healing, or to scientific biomedicine. In addition to seeking help from a biomedical or scientific health care provider, patients may also seek help from folk or religious healers.

  1. An elderly Mexican-American woman with traditional beliefs has been admitted to an inpatient care unit. A culturally-sensitive nurse would:

A) contact the hospital administrator about the best course of action.
B) automatically get a curandero for her because it is not culturally appropriate for her to request one.
C) further assess the patient’s cultural beliefs and offer the patient assistance in contacting a curandero or priest if she desires.
D) ask the family what they would like to do because Mexican-Americans traditionally give control of decisions to their families.
C) further assess the patient’s cultural beliefs and offer the patient assistance in contacting a curandero or priest if she desires.

Pages: 22-23 In addition to seeking help from the biomedical/scientific health care provider, patients may also seek help from folk or religious healers. Some people, such as those of Mexican-American or American Indian origins, may believe that the cure is incomplete unless the body, mind, and spirit are also healed (although the division of the person into parts is a Western concept).

  1. The nurse is reviewing concepts of cultural aspects of pain. Which statement is true regarding pain?

A) All patients will behave the same way when in pain.
B) Just as patients vary in their perceptions of pain, so will they vary in their expressions of pain.
C) Cultural norms have very little to do with pain tolerance, because pain tolerance is always biologically determined.
D) A patient’s expression of pain is largely dependent on the amount of tissue injury associated with the pain.
B) Just as patients vary in their perceptions of pain, so will they vary in their expressions of pain.

Page: 25 In addition to expecting variations in pain perception and tolerance, the nurse should expect variations in the expression of pain. It is well known that individuals turn to their social environment for validation and comparison. The other statements are incorrect.

  1. The nurse recognizes that working with children with a different cultural perspective may be especially difficult because:

A) children have spiritual needs that are influenced by their stages of development.
B) children have spiritual needs that are direct reflections of what is occurring in their homes.
C) religious beliefs rarely affect the parents’ perceptions of the illness.
D) parents are often the decision makers, and they have no knowledge of their children’s spiritual needs.
A) children have spiritual needs that are influenced by their stages of development.

Page: 20. Illness during childhood may be an especially difficult clinical situation. Children, as well as adults, have spiritual needs that vary according to the child’s developmental level and the religious climate that exists in the family. The other statements are not correct.

  1. When providing culturally competent care, nurses must incorporate cultural assessments into their health assessments. Which statement is most appropriate to use when initiating an assessment of cultural beliefs with an elderly American Indian patient?

A) “Are you of the Christian faith?”
B) “Do you want to see a medicine man?”
C) “How often do you seek help from medical providers?”
D) “What cultural or spiritual beliefs are important to you?”
D) “What cultural or spiritual beliefs are important to you?”

Page: 17. The nurse needs to assess the cultural beliefs and practices of the patient. American Indians may seek assistance from a medicine man or shaman, but the nurse should not assume this. An open-ended question regarding cultural and spiritual beliefs is best used initially when performing a cultural assessment.

  1. When planning a cultural assessment, the nurse should include which component?

A) Family history
B) Chief complaint
C) Medical history
D) Health-related beliefs
D) Health-related beliefs

Pages: 19-20. Health-related beliefs and practices are one component of a cultural assessment. The other items reflect other aspects of the patient’s history.

  1. When the nurse is evaluating the reliability of a patient’s responses, which of these statements would be correct? The patient:

A. has a history of drug abuse and therefore is not reliable.
B. provided consistent information and therefore is reliable.
C. smiled throughout interview and therefore is assumed reliable.
D. would not answer questions concerning stress and therefore is not reliable.
B. provided consistent information and therefore is reliable.

Page: 50. A reliable person always gives the same answers, even when questions are rephrased or are repeated later in the interview. The other statements are not correct.

  1. In recording the childhood illnesses of a patient who denies having had any, which note by the nurse would be most accurate?

A. Patient denies usual childhood illnesses.
B. Patient states he was a “very healthy” child.
C. Patient states sister had measles, but he didn’t.
D. Patient denies measles, mumps, rubella, chickenpox, pertussis, and strep throat.
D. Patient denies measles, mumps, rubella, chickenpox, pertussis, and strep throat.

Page: 51. Childhood illnesses include measles, mumps, rubella, chickenpox, pertussis, and strep throat. Avoid recording “usual childhood illnesses” because an illness common in the person’s childhood may be unusual today (e.g., measles).

  1. The mother of a 16-month-old toddler tells the nurse that her daughter has an earache. What would be an appropriate response?

A. “Maybe she is just teething.”
B. “I will check her ear for an ear infection.”
C. “Are you sure she is really having pain?”
D. “Please describe what she is doing to indicate she is having pain.”
D. “Please describe what she is doing to indicate she is having pain.”

Page: 60. With a very young child, ask the parent, “How do you know the child is in pain?” Pulling at ears alerts parent to ear pain. The statements about teething and questioning whether the child is really having pain do not explore the symptoms, which should be done before a physical examination.

  1. A 5-year-old boy is being admitted to the hospital to have his tonsils removed. Which information should the nurse collect before this procedure?

A. The child’s birth weight
B. The age at which he crawled
C. Whether he has had the measles
D. Reactions to previous hospitalizations
D. Reactions to previous hospitalizations

Assess how the child reacted to hospitalization and any complications. If the child reacted poorly, he or she may be afraid now and will need special preparation for the examination that is to follow. The other items are not significant for the procedure.

  1. The nurse is preparing to do a functional assessment. Which statement best describes the purpose of a functional assessment?

A. It assesses how the individual is coping with life at home.
B. It determines how children are meeting developmental milestones.
C. It can identify any problems with memory the individual may be experiencing.
D. It helps to determine how a person is managing day-to-day activities.
D. It helps to determine how a person is managing day-to-day activities.

Page: 67. The functional assessment measures how a person manages day-to-day activities. The other answers do not reflect the purpose of a functional assessment.

  1. The nurse is performing a functional assessment on an 82-year-old patient who recently had a stroke. Which of these questions would be most important to ask?

A. “Do you wear glasses?”
B. “Are you able to dress yourself?”
C. “Do you have any thyroid problems?”
D. “How many times a day do you have a bowel movement?”
B. “Are you able to dress yourself?”

Page: 67. Functional assessment measures how a person manages day-to-day activities. For the older person, the meaning of health becomes those activities that they can or cannot do. The other responses do not relate to functional assessment.

  1. The nurse is conducting a developmental history on a 5-year-old child. Which questions are appropriate to ask the parents for this part of the assessment? Select all that apply.

A. “How much junk food does your child eat?”
B. “How many teeth has he lost, and when did he lose them?”
C. “Is he able to tie his shoelaces?”
D. “Does he take a children’s vitamin?”
E. “Can he tell time?”
F. “Does he have any food allergies?”
B. “How many teeth has he lost, and when did he lose them?”
C. “Is he able to tie his shoelaces?”
E. “Can he tell time?”

Page: 61. Questions about tooth loss, ability to tell time, and ability to tie shoelaces are appropriate questions for a developmental assessment. Questions about junk food intake and vitamins are part of a nutritional history. Questions about food allergies are not part of a developmental history.

  1. During an examination, the nurse can assess mental status by which activity?

A) Examining the patient’s electroencephalogram
B) Observing the patient as he or she performs an IQ test
C) Observing the patient and inferring health or dysfunction
D) Examining the patient’s response to a specific set of questions
C) Observing the patient and inferring health or dysfunction

Page: 71. Mental status cannot be scrutinized directly like the characteristics of skin or heart sounds. Its functioning is inferred through assessment of an individual’s behaviors, such as consciousness, language, mood and affect, and other aspects.

  1. The nurse is assessing a 75-year-old man. As the nurse begins the mental status portion of the assessment, the nurse expects that this patient:

A) will have no decrease in any of his abilities, including response time.
B) will have difficulty on tests of remote memory because this typically decreases with age.
C) may take a little longer to respond, but his general knowledge and abilities should not have declined.
D) will have had a decrease in his response time because of language loss and a decrease in general knowledge.
C) may take a little longer to respond, but his general knowledge and abilities should not have declined.

Page: 72. The aging process leaves the parameters of mental status mostly intact. There is no decrease in general knowledge and little or no loss in vocabulary. Response time is slower than in youth. It takes a bit longer for the brain to process information and to react to it. Recent memory, which requires some processing is somewhat decreased with aging, but remote memory is not affected.

  1. The nurse is preparing to do a mental status examination. Which statement is true regarding the mental status examination?

A) A patient’s family is the best resource for information about the patient’s coping skills.
B) It is usually sufficient to gather mental status information during the health history interview.
C) It takes an enormous amount of extra time to integrate the mental status examination into the health history interview.
D) It is usually necessary to perform a complete mental status examination to get a good idea of the patient’s level of functioning.
B) It is usually sufficient to gather mental status information during the health history interview.

Page: 73. The full mental status examination is a systematic check of emotional and cognitive functioning. The steps described here, though, rarely need to be taken in their entirety. Usually, one can assess mental status through the context of the health history interview.

  1. During a mental status examination, the nurse wants to assess a patient’s affect. The nurse should ask the patient which question?

A) “How do you feel today?”
B) “Would you please repeat the following words?”
C) “Have these medications had any effect on your pain?”
D) “Has this pain affected your ability to get dressed by yourself?”
A) “How do you feel today?”

Page: 74. Judge mood and affect by body language and facial expression and by asking directly, “How do you feel today?” or “How do you usually feel?” The mood should be appropriate to the person’s place and condition and should change appropriately with topics.

  1. During a mental status assessment, which question by the nurse would best assess a person’s judgment?

A) “Do you feel that you are being watched, followed, or controlled?”
B) “Tell me about what you plan to do once you are discharged from the hospital.”
C) “What does the statement, ‘People in glass houses shouldn’t throw stones,’ mean to you?”
D) “What would you do if you found a stamped, addressed envelope lying on the sidewalk?”
B) “Tell me about what you plan to do once you are discharged from the hospital.”

Pages: 76-77. A person exercises judgment when he or she can compare and evaluate the alternatives in a situation and reach an appropriate course of action. Rather than testing the person’s response to a hypothetical situation (as illustrated in the option with the envelope), the nurse should be more interested in the person’s judgment about daily or long-term goals, the likelihood of acting in response to delusions or hallucinations and the capacity for violent or suicidal behavior.

  1. The nurse is performing a mental status examination. Which statement is true regarding the assessment of mental status?

A) Mental status assessment diagnoses specific psychiatric disorders.
B) Mental disorders occur in response to everyday life stressors.
C) Mental status functioning is inferred through assessment of an individual’s behaviors.
D) Mental status can be assessed directly, just like other systems of the body (e.g., cardiac and breath sounds).
C) Mental status functioning is inferred through assessment of an individual’s behaviors.

Page: 71. Mental status functioning is inferred through assessment of an individual’s behaviors. It cannot be assessed directly like characteristics of the skin or heart sounds.

  1. When performing a physical assessment, the technique the nurse will always use first is:

A) palpation.
B) inspection.
C) percussion.
D) auscultation.
B) inspection.

Pages: 115-116. The skills requisite for the physical examination are inspection, palpation, percussion, and auscultation. The skills are performed one at a time and in this order (with the exception of the abdominal assessment, where auscultation takes place before palpation and percussion). The assessment of each body system begins with inspection. A focused inspection takes time and yields a surprising amount of information.

  1. The nurse is assessing a patient’s skin during an office visit. What is the best technique to use to best assess the patient’s skin temperature? Use the:

A) fingertips because they’re more sensitive to small changes in temperature.
B) dorsal surface of the hand because the skin is thinner than on the palms.
C) ulnar portion of the hand because there is increased blood supply that enhances temperature sensitivity.
D) palmar surface of the hand because it is most sensitive to temperature variations because of increased nerve supply in this area.
B) dorsal surface of the hand because the skin is thinner than on the palms.

The dorsa (backs) of hands and fingers are best for determining temperature because the skin there is thinner than on the palms. Fingertips are best for fine, tactile discrimination; the other responses are not useful for palpation.

  1. The nurse is preparing to assess a patient’s abdomen by palpation. How should the nurse proceed?

A) Avoid palpation of reported “tender” areas because this may cause the patient pain.
B) Quickly palpate a tender area to avoid any discomfort that the patient may experience.
C) Begin the assessment with deep palpation, encouraging the patient to relax and take deep breaths.
D) Start with light palpation to detect surface characteristics and to accustom the patient to being touched.
D) Start with light palpation to detect surface characteristics and to accustom the patient to being touched.

Pages: 115-116. Light palpation is performed initially to detect any surface characteristics and to accustom the person to being touched. Tender areas should be palpated last, not first.

  1. The nurse would use bimanual palpation technique in which situation?

A) Palpating the thorax of an infant
B) Palpating the kidneys and uterus
C) Assessing pulsations and vibrations
D) Assessing the presence of tenderness and pain
B) Palpating the kidneys and uterus

Pages: 115-116. Bimanual palpation requires the use of both hands to envelop or capture certain body parts or organs such as the kidneys, uterus, or adnexa. The other situations are not appropriate for bimanual palpation.

  1. When percussing over the liver of a patient, the nurse notices a dull sound. The nurse should:

A) consider this a normal finding.
B) palpate this area for an underlying mass.
C) reposition the hands and attempt to percuss in this area again.
D) consider this an abnormal finding and refer the patient for additional treatment.
A) consider this a normal finding.

Pages: 116-117. Percussion over relatively dense organs, such as the liver or spleen, will produce a dull sound. The other responses are not correct.

  1. The nurse is unable to palpate the right radial pulse on a patient. The best action would be to:

A) auscultate over the area with a fetoscope.
B) use a goniometer to measure the pulsations.
C) use a Doppler device to check for pulsations over the area.
D) check for the presence of pulsations with a stethoscope.
C) use a Doppler device to check for pulsations over the area.

Page: 120. Doppler devices are used to augment pulse or blood pressure measurements. Goniometers measure joint range of motion. A fetoscope is used to auscultate fetal heart tones. Stethoscopes are used to auscultate breath, bowel, and heart sounds.

  1. When performing a physical examination, safety must be considered to protect the examiner and the patient against the spread of infection. Which of these statements describes the most appropriate action the nurse should take when performing a physical examination?

A) There is no need to wash one’s hands after removing gloves, as long as the gloves are still intact.
B) Wash hands before and after every physical patient encounter.
C) Wash hands between the examination of each body system to prevent the spread of bacteria from one part of the body to another.
D) Wear gloves throughout the entire examination to demonstrate to the patient concern regarding the spread of infectious diseases.
B) Wash hands before and after every physical patient encounter.

Page: 120. The nurse should wash his or her hands before and after every physical patient encounter; after contact with blood, body fluids, secretions, and excretions; after contact with any equipment contaminated with body fluids; and after removing gloves. Hands should be washed after gloves have been removed, even if the gloves appear to be intact. Gloves should be worn when there is potential contact with any body fluids.

  1. The nurse is examining an infant and prepares to elicit the Moro reflex at which time during the examination?

A) When the infant is sleeping
B) At the end of the examination
C) Before auscultation of the thorax
D) Halfway through the examination
B) At the end of the examination

Page: 123. Elicit the Moro or “startle” reflex at the end of the examination because it may cause the infant to cry.

  1. A 6-month-old infant has been brought to the well-child clinic for a check-up. She is currently sleeping. What should the nurse do first when beginning the examination?

A) Auscultate the lungs and heart while the infant is still sleeping.
B) Examine the infant’s hips because this procedure is uncomfortable.
C) Begin with the assessment of the eye and continue with the remainder of the examination in a head-to-toe approach.
D) Wake the infant before beginning any portion of the examination to obtain the most accurate assessment of body systems.
A) Auscultate the lungs and heart while the infant is still sleeping.

Pages: 122-124. When the infant is quiet or sleeping is an ideal time to assess the cardiac, respiratory, and abdominal systems. Assessment of the eye, ear, nose, and throat are invasive procedures and should be performed at the end of the examination.

  1. The nurse is assessing an 80-year-old male patient. Which assessment findings would be considered normal?

A) An increase in body weight from younger years
B) Additional deposits of fat on the thighs and lower legs
C) The presence of kyphosis and flexion in the knees and hips
D) A change in overall body proportion, a longer trunk, and shorter extremities
C) The presence of kyphosis and flexion in the knees and hips

Page: 149. Changes that occur in the aging person include more prominent bony landmarks, decreased body weight (especially in males), a decrease in subcutaneous fat from the face and periphery, and additional fat deposited on the abdomen and hips. Postural changes of kyphosis and slight flexion in the knees and hips also occur.

  1. When assessing the force, or strength, of a pulse, the nurse recalls that it:

A) is usually recorded on a 0- to 2-point scale.
B) demonstrates elasticity of the vessel wall.
C) is a reflection of the heart’s stroke volume.
D) reflects the blood volume in the arteries during diastole.
C) is a reflection of the heart’s stroke volume.

Page: 134. The heart pumps an amount of blood (the stroke volume) into the aorta. The force flares the arterial walls and generates a pressure wave, which is felt in the periphery as the pulse.

  1. When assessing the quality of a patient’s pain, the nurse should ask which question?

A) “When did the pain start?”
B) “Is the pain a stabbing pain?”
C) “Is it a sharp pain or dull pain?”
D) “What does your pain feel like?”
D) “What does your pain feel like?”

Page: 164. To assess the quality of a person’s pain, have the patient describe the pain in his or her own words.

  1. The nurse is providing care for a 68-year-old woman who is complaining of constipation. What concern exists regarding her nutritional status?

A) The absorption of nutrients may be impaired.
B) The constipation may represent a food allergy.
C) She may need emergency surgery for the problem.
D) The gastrointestinal problem will increase her caloric demand.
A) The absorption of nutrients may be impaired.

Page: 182. Gastrointestinal symptoms such as vomiting, diarrhea, or constipation may interfere with nutrient intake or absorption. The other responses are not correct.

  1. During a nutritional assessment, why is it important for the nurse to ask a patient what medications he or she is taking?

A) Certain drugs can affect the metabolism of nutrients.
B) The nurse needs to assess the patient for allergic reactions.
C) Medications need to be documented on the record for the physician’s review.
D) Medications can affect one’s memory and ability to identify food eaten in the last 24 hours.
A) Certain drugs can affect the metabolism of nutrients.

Page: 183
Analgesics, antacids, anticonvulsants, antibiotics, diuretics, laxatives, antineoplastic drugs, steroids, and oral contraceptives are drugs that can interact with nutrients, impairing their digestion, absorption, metabolism, or use. The other responses are not correct.

  1. The nurse is reviewing the nutritional assessment of an 82-year-old patient. Which of these factors is most likely to affect the nutritional status of an elderly person?

A) Increase in taste and smell
B) Living alone on a fixed income
C) Change in cardiovascular status
D) Increase in gastrointestinal motility and absorption
B) Living alone on a fixed income

Page: 176. Socioeconomic conditions frequently affect the nutritional status of the aging adult; these factors should be closely evaluated. Physical limitations, income, and social isolation are frequent problems that interfere with the acquisition of a balanced diet. A decrease in taste and smell and decreased gastrointestinal motility and absorption occur with aging. Cardiovascular status is not a factor that affects an elderly person’s nutritional status.

  1. When the mid-upper arm circumference and triceps skinfold of an 82-year-old man are evaluated, which is important for the nurse to remember?

A) These measurements are no longer necessary for the elderly.
B) Derived weight measures may be difficult to interpret because of wide ranges of normal.
C) These measurements may not be accurate because of changes in skin and fat distribution.
D) Measurements may be difficult to obtain if the patient is unable to flex his elbow to at least 90 degrees.
C) These measurements may not be accurate because of changes in skin and fat distribution.

Page: 191
Accurate mid-upper arm circumference and triceps skinfold measurements are difficult to obtain and interpret in older adults because of sagging skin, changes in fat distribution, and declining muscle mass. Body mass index and waist-to-hip ratio are better indicators of obesity in the elderly.

  1. The nurse needs to perform anthropometric measures of an 80-year-old man who is confined to a wheelchair. Which of the following is true in this situation?

A) Changes in fat distribution will affect the waist-to-hip ratio.
B) Height measurements may not be accurate because of changes in bone.
C) Declining muscle mass will affect the triceps skinfold measure.
D) Mid-arm circumference is difficult to obtain because of loss of skin elasticity.
B) Height measurements may not be accurate because of changes in bone.

Page: 191. Height measures may not be accurate in individuals confined to a bed or wheelchair or those over 60 years of age because of osteoporotic changes.

  1. The nurse is performing a nutritional assessment on an 80-year-old patient. The nurse knows that physiological changes that directly affect the nutritional status of the elderly include:

A) slowed gastrointestinal motility.
B) hyperstimulation of the salivary glands.
C) an increased sensitivity to spicy and aromatic foods.
D) decreased gastrointestinal absorption causing esophageal reflux.
A) slowed gastrointestinal motility.

Page: 176. Normal physiological changes in aging adults that affect nutritional status include slowed gastrointestinal motility, decreased gastrointestinal absorption, diminished olfactory and taste sensitivity, decreased saliva production, decreased visual acuity, and poor dentition.

  1. The nurse keeps in mind that a thorough skin assessment is very important because the skin holds information about a person’s:

A) support systems.
B) circulatory status.
C) socioeconomic status.
D) psychological wellness.
B) circulatory status.

Page: 211. The skin holds information about the body’s circulation, nutritional status, and signs of systemic diseases as well as topical data on the integument itself.

  1. A patient tells the nurse that he has noticed that one of his moles has started to burn and bleed. When assessing his skin, the nurse would pay special attention to the danger signs for pigmented lesions and would be concerned with which additional finding?

A) Color variation
B) Border regularity
C) Symmetry of lesions
D) Diameter less than 6 mm
A) Color variation

Pages: 212-213. Abnormal characteristics of pigmented lesions are summarized in the mnemonic ABCD: asymmetry of pigmented lesion, border irregularity, color variation, and diameter greater than 6 mm.

  1. An elderly woman is brought to the emergency department after being found lying on the kitchen floor 2 days, and she is extremely dehydrated. What would the nurse expect to see upon examination?

A) Smooth mucous membranes and lips
B) Dry mucous membranes and cracked lips
C) Pale mucous membranes
D) White patches on the mucous membranes
B) Dry mucous membranes and cracked lips

Page: 215. With dehydration, mucous membranes look dry and lips look parched and cracked. The other responses are not found in dehydration.

  1. A 65-year-old man with emphysema and bronchitis has come to the clinic for a follow-up appointment. On assessment, the nurse might expect to see which assessment finding?

A) Anasarca
B) Scleroderma
C) Pedal erythema
D) Clubbing of the nails
D) Clubbing of the nails

Pages: 217-218. Clubbing of the nails occurs with congenital cyanotic heart disease, neoplastic, and pulmonary diseases. The other responses are assessment findings not associated with pulmonary diseases.

  1. The nurse has discovered decreased skin turgor in a patient and knows that this is an expected finding in which of these conditions?

A) Severe obesity
B) Childhood growth spurts
C) Severe dehydration
D) Connective tissue disorders such as scleroderma
C) Severe dehydration

Page: 215. Decreased skin turgor is associated with severe dehydration or extreme weight loss.

  1. A 40-year-old woman reports a change in mole size, accompanied by color changes, itching, burning, and bleeding over the past month. She has a dark complexion and has no family history of skin cancer, but she has had many blistering sunburns in the past. The nurse would:

A) tell the patient to watch the lesion and report back in 2 months.
B) refer the patient because of the suspicion of melanoma on the basis of her symptoms.
C) ask additional questions regarding environmental irritants that may have caused this condition.
D) suspect that this is a compound nevus, which is very common in young to middle-aged adults.
B) refer the patient because of the suspicion of melanoma on the basis of her symptoms.

The ABCD danger signs of melanoma are asymmetry, border irregularity, color variation, and diameter. In addition, individuals may report a change in size, development of itching, burning, bleeding, or a new-pigmented lesion. Any of these signs raise suspicion of malignant melanoma and warrant immediate referral.

  1. The nurse is assessing for clubbing of the fingernails and would expect to find:

A) a nail base that is firm and slightly tender.
B) curved nails with a convex profile and ridges across the nail.
C) a nail base that feels spongy with an angle of the nail base of 150 degrees.
D) an angle of the nail base of 180 degrees or greater with a nail base that feels spongy.
D) an angle of the nail base of 180 degrees or greater with a nail base that feels spongy.

Pages: 217-218. The normal nail is firm at its base and has an angle of 160 degrees. In clubbing, the angle straightens to 180 degrees or greater and the nail base feels spongy.

  1. A patient has been admitted for severe psoriasis. The nurse can expect to see what finding in the patient’s fingernails?

A) Splinter hemorrhages
B) Paronychia
C) Pitting
D) Beau lines
C) Pitting

Pages: 248-250. Pitting nails are characterized by sharply defined pitting and crumbling of the nails with distal detachment, and they are associated with psoriasis. See Table 12-13 for descriptions of the other terms.

  1. The nurse suspects that a patient has hyperthyroidism and laboratory data indicate that the patient’s thyroxine and tri-iodothyronine hormone levels are elevated. Which of these findings would the nurse most likely find on examination?

A) Tachycardia
B) Constipation
C) Rapid dyspnea
D) Atrophied nodular thyroid
A) Tachycardia

Thyroxine and tri-iodothyronine are thyroid hormones that stimulate the rate of cellular metabolism, resulting in tachycardia. With an enlarged thyroid as in hyperthyroidism, the nurse might expect to find diffuse enlargement (goiter) or a nodular lump, but not an atrophied gland. Dyspnea and constipation are not findings associated with hyperthyroidism.

  1. During an examination, the nurse knows that Paget’s disease would be indicated by which of these assessment findings?

A) Positive Macewen sign
B) Premature closure of the sagittal suture
C) Headache, vertigo, tinnitus, and deafness
D) Elongated head with heavy eyebrow ridge
C) Headache, vertigo, tinnitus, and deafness

Paget’s disease occurs more often in males and is characterized by bowed long bones, sudden fractures, and enlarging skull bones that press on cranial nerves causing symptoms of headache, vertigo, tinnitus, and progressive deafness.

  1. A woman comes to the clinic and states, “I’ve been sick for so long! My eyes have gotten so puffy, and my eyebrows and hair have become coarse and dry.” The nurse will assess for other signs and symptoms of:

A) cachexia.
B) Parkinson’s syndrome.
C) myxedema.
D) scleroderma.
C) myxedema.

Pages: 276-277. Myxedema (hypothyroidism) is a deficiency of thyroid hormone that, when severe, causes a nonpitting edema or myxedema. The patient will have a puffy edematous face especially around eyes (periorbital edema), coarse facial features, dry skin, and dry, coarse hair and eyebrows. See Table 13-4, Abnormal Facial Appearances with Chronic Illnesses, for descriptions of the other responses.

  1. The physician reports that a patient with a neck tumor has a tracheal shift. The nurse is aware that this means that the patient’s trachea is:

A) pulled to the affected side.
B) pushed to the unaffected side.
C) pulled downward.
D) pulled downward in a rhythmic pattern.
B) pushed to the unaffected side.

Pages: 262-263. The trachea is pushed to the unaffected side with an aortic aneurysm, a tumor, unilateral thyroid lobe enlargement, and pneumothorax. The trachea is pulled to the affected side with large atelectasis, pleural adhesions, or fibrosis. Tracheal tug is a rhythmic downward pull that is synchronous with systole and occurs with aortic arch aneurysm.

  1. During an assessment of an infant, the nurse notes that the fontanels are depressed and sunken. The nurse suspects which condition?

A) Rickets
B) Dehydration
C) Mental retardation
D) Increased intracranial pressure
B) Dehydration

Pages: 265-266. Depressed and sunken fontanels occur with dehydration or malnutrition. Mental retardation and rickets have no effect on fontanels. Increased intracranial pressure would cause tense or bulging, and possibly pulsating fontanels.

  1. The nurse is performing an assessment on a 7-year-old child who has symptoms of chronic watery eyes, sneezing, and clear nasal drainage. The nurse notices the presence of a transverse line across the bridge of the nose, dark blue shadows below the eyes, and a double crease on the lower eyelids. These findings are characteristic of:

A) allergies.
B) a sinus infection.
C) nasal congestion.
D) an upper respiratory infection.
A) allergies.

Page: 275. Chronic allergies often develop chronic facial characteristics. These include blue shadows below the eyes, a double or single crease on the lower eyelids, open-mouth breathing, and a transverse line on the nose.

  1. A mother asks when her newborn infant’s eyesight will be developed. The nurse should reply:

A) “Vision is not totally developed until 2 years of age.”
B) “Infants develop the ability to focus on an object at around 8 months.”
C) “By about 3 months, infants develop more coordinated eye movements and can fixate on an object.”
D) “Most infants have uncoordinated eye movements for the first year of life.”
C) “By about 3 months, infants develop more coordinated eye movements and can fixate on an object.”

Page: 284. Eye movements may be poorly coordinated at birth, but by 3 to 4 months of age, the infant should establish binocularity and should be able to fixate on a single image with both eyes simultaneously.

  1. The nurse is performing an eye assessment on an 80-year-old patient. Which of these findings is considered abnormal?

A) A decrease in tear production
B) Unequal pupillary constriction in response to light
C) The presence of arcus senilis seen around the cornea
D) Loss of the outer hair on the eyebrows due to a decrease in hair follicles
B) Unequal pupillary constriction in response to light

Pages: 305-308. Pupils are small in old age, and the pupillary light reflex may be slowed, but pupillary constriction should be symmetric. The assessment findings in the other responses are considered normal in older persons.

  1. The nurse notices the presence of periorbital edema when performing an eye assessment on a 70-year-old patient. The nurse should:

A) check for the presence of exophthalmos.
B) suspect that the patient has hyperthyroidism.
C) ask the patient if he or she has a history of heart failure.
D) assess for blepharitis because this is often associated with periorbital edema.
C) ask the patient if he or she has a history of heart failure.

Page: 312. Periorbital edema occurs with local infections, crying, and systemic conditions such as heart failure, renal failure, allergy, and hypothyroidism. Periorbital edema is not associated with blepharitis.

  1. A patient comes to the emergency department after a boxing match, and his left eye is swollen almost shut. He has bruises on his face and neck. He says he is worried because he “can’t see well” from his left eye. The physician suspects retinal damage. The nurse recognizes that signs of retinal detachment include:

A) loss of central vision.
B) shadow or diminished vision in one quadrant or one half of the visual field.
C) loss of peripheral vision.
D) sudden loss of pupillary constriction and accommodation.
B) shadow or diminished vision in one quadrant or one half of the visual field.

Page: 316. With retinal detachment, the person has shadows or diminished vision in one quadrant or one half of the visual field. The other responses are not signs of retinal detachment.

  1. A 68-year-old woman is in the eye clinic for a checkup. She tells the nurse that she has been having trouble with reading the paper, sewing, and even seeing the faces of her grandchildren. On examination, the nurse notes that she has some loss of central vision but her peripheral vision is normal. These findings suggest that:

A) she may have macular degeneration.
B) her vision is normal for someone her age.
C) she has the beginning stages of cataract formation.
D) she has increased intraocular pressure or glaucoma.
A) she may have macular degeneration.

Page: 285. Macular degeneration is the most common cause of blindness. It is characterized by loss of central vision. Cataracts would show lens opacity. Chronic open-angle glaucoma, the most common type of glaucoma, involves a gradual loss of peripheral vision.

  1. An ophthalmic examination reveals papilledema. The nurse is aware that this finding indicates:

A) retinal detachment.
B) diabetic retinopathy.
C) acute-angle glaucoma.
D) increased intracranial pressure.
D) increased intracranial pressure.

Pages: 319-320. Papilledema, or choked disk, is a serious sign of increased intracranial pressure, which is caused by a space-occupying mass such as a brain tumor or hematoma. This pressure causes venous stasis in the globe, showing redness, congestion, and elevation of the optic disc, blurred margins, hemorrhages, and absent venous pulsations. Papilledema is not associated with the conditions in the other responses.

  1. During an examination, a patient states that she was diagnosed with open-angle glaucoma 2 years ago. The nurse assesses for characteristics of open-angle glaucoma. Which of these are characteristics of open-angle glaucoma? Select all that apply.

A) The patient may experience sensitivity to light, nausea, and halos around lights.
B) The patient experiences tunnel vision in late stages.
C) Immediate treatment is needed.
D) Vision loss begins with peripheral vision.
E) It causes sudden attacks of increased pressure that cause blurred vision.
F) There are virtually no symptoms.
B) The patient experiences tunnel vision in late stages.
D) Vision loss begins with peripheral vision.
F) There are virtually no symptoms.

Pages: 308-309. Open-angle glaucoma is the most common type of glaucoma; there are virtually no symptoms. Vision loss begins with the peripheral vision, which often goes unnoticed because individuals learn to compensate intuitively by turning their heads. The other characteristics are those of closed-angle glaucoma.

  1. The nurse is taking the history of a patient who may have a perforated eardrum. What would be an important question in this situation?

A) “Do you ever notice ringing or crackling in your ears?”
B) “When was the last time you had your hearing checked?”
C) “Have you ever been told you have any type of hearing loss?”
D) “Was there any relationship between the ear pain and the discharge you mentioned?”
D) “Was there any relationship between the ear pain and the discharge you mentioned?”

Pages: 327-328. Typically with perforation, ear pain occurs first, stopping with a popping sensation, and then drainage occurs.

  1. The nurse is performing an ear examination of an 80-year-old patient. Which of these would be considered a normal finding?

A) A high-tone frequency loss
B) Increased elasticity of the pinna
C) A thin, translucent membrane
D) A shiny, pink tympanic membrane
A) A high-tone frequency loss

Pages: 337-338. A high-tone frequency hearing loss is apparent for those affected with presbycusis, the hearing loss that occurs with aging. The pinna loses elasticity, causing earlobes to be pendulous. The eardrum may be whiter in color and more opaque and duller than in the young adult.

  1. During an examination, the patient states he is hearing a buzzing sound and says that it is “driving me crazy!” The nurse recognizes that this symptom indicates:

A) vertigo.
B) pruritus.
C) tinnitus.
D) cholesteatoma.
C) tinnitus.

Pages: 328-329. Tinnitus is a sound that comes from within a person; it can be a ringing, crackling, or buzzing sound. It accompanies some hearing or ear disorders.

  1. The nurse is testing the hearing of a 78-year-old man and keeps in mind the changes in hearing that occur with aging include which of the following? Select all that apply.

A) Hearing loss related to aging begins in the mid 40s.
B) The progression is slow.
C) The aging person has low-frequency tone loss.
D) The aging person may find it harder to hear consonants than vowels.
E) Sounds may be garbled and difficult to localize.
F) Hearing loss reflects nerve degeneration of the middle ear.
B) The progression is slow.
D) The aging person may find it harder to hear consonants than vowels.
E) Sounds may be garbled and difficult to localize.

Page: 326. Presbycusis is a type of hearing loss that occurs with aging and is found in 60% of those older than 65 years. It is a gradual sensorineural loss caused by nerve degeneration in the inner ear or auditory nerve, and it slowly progresses after age 50. The person first notices a high-frequency tone loss; it is harder to hear consonants (high-pitched components of speech) than vowels. This makes words sound garbled. The ability to localize sound is impaired also.

  1. When assessing a patient’s lungs, the nurse recalls that the left lung:

A) consists of two lobes.
B) is divided by the horizontal fissure.
C) consists primarily of an upper lobe on the posterior chest.
D) is shorter than the right lung because of the underlying stomach.
A) consists of two lobes.

Pages: 413-414. The left lung has two lobes, and the right lung has three lobes. The right lung is shorter than the left lung because of the underlying liver. The left lung is narrower than the right lung because the heart bulges to the left. The posterior chest is almost all lower lobe.

  1. During an assessment, the nurse knows that expected assessment findings in the normal adult lung include the presence of:

A) adventitious sounds and limited chest expansion.
B) increased tactile fremitus and dull percussion tones.
C) muffled voice sounds and symmetrical tactile fremitus.
D) absent voice sounds and hyperresonant percussion tones.
C) muffled voice sounds and symmetrical tactile fremitus.

Pages: 429-430. Normal lung findings include symmetric chest expansion, resonant percussion tones, vesicular breath sounds over the peripheral lung fields, muffled voice sounds, and no adventitious sounds.

  1. A 65-year-old patient with a history of heart failure comes to the clinic with complaints of “being awakened from sleep with shortness of breath.” Which action by the nurse is most appropriate?

A) Obtain a detailed history of the patient’s allergies and history of asthma.
B) Tell the patient to sleep on his or her right side to facilitate ease of respirations.
C) Assess for other signs and symptoms of paroxysmal nocturnal dyspnea.
D) Assure the patient that this is normal and will probably resolve within the next week.
C) Assess for other signs and symptoms of paroxysmal nocturnal dyspnea.

Pages: 419-420. The patient is experiencing paroxysmal nocturnal dyspnea: being awakened from sleep with shortness of breath and the need to be upright to achieve comfort.

  1. When assessing tactile fremitus, the nurse recalls that it is normal to feel tactile fremitus most intensely over which location?

A) Between the scapulae
B) Third intercostal space, MCL
C) Fifth intercostal space, MAL
D) Over the lower lobes, posterior side
A) Between the scapulae

Page: 424. Normally, fremitus is most prominent between the scapulae and around the sternum. These are sites where the major bronchi are closest to the chest wall. Fremitus normally decreases as one progress down the chest because more tissue impedes sound transmission.

  1. The nurse is reviewing the technique of palpating for tactile fremitus with a new graduate. Which statement by the graduate nurse reflects a correct understanding of tactile fremitus? “Tactile fremitus:

A) is caused by moisture in the alveoli.”
B) indicates that there is air in the subcutaneous tissues.”
C) is caused by sounds generated from the larynx.”
D) reflects the blood flow through the pulmonary arteries.”
C) is caused by sounds generated from the larynx.”

Pages: 422-423. Fremitus is a palpable vibration. Sounds generated from the larynx are transmitted through patent bronchi and the lung parenchyma to the chest wall where they are felt as vibrations. Crepitus is the term for air in the subcutaneous tissues.

  1. When auscultating the lungs of an adult patient, the nurse notes that over the posterior lower lobes low-pitched, soft breath sounds are heard, with inspiration being longer than expiration. The nurse interprets that these are:

A) sounds normally auscultated over the trachea.
B) bronchial breath sounds and are normal in that location.
C) vesicular breath sounds and are normal in that location.
D) bronchovesicular breath sounds and are normal in that location.
C) vesicular breath sounds and are normal in that location.

Pages: 428-429. Vesicular breath sounds are low-pitched, soft sounds with inspiration being longer than expiration. These breath sounds are expected over peripheral lung fields where air flows through smaller bronchioles and alveoli.

  1. The nurse is percussing over the lungs of a patient with pneumonia. The nurse knows that percussion over an area of atelectasis in the lungs would reveal:

A) dullness.
B) tympany.
C) resonance.
D) hyperresonance.
A) dullness.

Pages: 424-425. A dull percussion note signals an abnormal density in the lungs, as with pneumonia, pleural effusion, atelectasis, or tumor.

  1. The nurse knows that auscultation of fine crackles would most likely be noticed in:

A) a healthy 5-year-old child.
B) a pregnant woman.
C) the immediate newborn period.
D) association with a pneumothorax.
C) the immediate newborn period.

Pages: 436-437. Fine crackles are commonly heard in the immediate newborn period as a result of the opening of the airways and clearing of fluid. Persistent fine crackles would be noticed with pneumonia, bronchiolitis, or atelectasis.

  1. During auscultation of the lungs of an adult patient, the nurse notices the presence of bronchophony. The nurse should assess for signs of which condition?

A) Airway obstruction
B) Emphysema
C) Pulmonary consolidation
D) Asthma
C) Pulmonary consolidation

Page: 446. Pathologic conditions that increase lung density, such as pulmonary consolidation, will enhance transmission of voice sounds, such as bronchophony. See Table 18-7.

  1. The nurse is listening to the breath sounds of a patient with severe asthma. Air passing through narrowed bronchioles would produce which of these adventitious sounds?

A) Wheezes
B) Bronchial sounds
C) Bronchophony
D) Whispered pectoriloquy
A) Wheezes

Page: 445. Wheezes are caused by air squeezed or compressed through passageways narrowed almost to closure by collapsing, swelling, secretions, or tumors, such as with acute asthma or chronic emphysema.

  1. An adult patient with a history of allergies comes to the clinic complaining of wheezing and difficulty in breathing when working in his yard. The assessment findings include tachypnea, use of accessory neck muscles, prolonged expiration, intercostal retractions, decreased breath sounds, and expiratory wheezes. The nurse interprets that these assessment findings are consistent with:

A) asthma.
B) atelectasis.
C) lobar pneumonia.
D) heart failure.
A) asthma.

Page: 451. Asthma is allergic hypersensitivity to certain inhaled particles that produces inflammation and a reaction of bronchospasm, which increases airway resistance, especially during expiration. Increased respiratory rate, use of accessory muscles, retraction of intercostal muscles, prolonged expiration, decreased breath sounds, and expiratory wheezing are all characteristic of asthma. See Table 18-8 for descriptions of the other conditions.

  1. During auscultation of breath sounds, the nurse should use the stethoscope correctly, in which of the following ways?

A) Listen to at least one full respiration in each location.
B) Listen as the patient inhales and then go to the next site during exhalation.
C) Have the patient breathe in and out rapidly while the nurse listens to the breath sounds.
D) If the patient is modest, listen to sounds over his or her clothing or hospital gown.
A) Listen to at least one full respiration in each location.

Pages: 426-427. During auscultation of breath sounds with a stethoscope, it is important to listen to one full respiration in each location. During the examination, the nurse should monitor the breathing and offer times for the person to breathe normally to prevent possible dizziness.

  1. During palpation of the anterior chest wall, the nurse notices a coarse, crackling sensation over the skin surface. On the basis of these findings, the nurse suspects:

A) tactile fremitus.
B) crepitus.
C) friction rub.
D) adventitious sounds.
B) crepitus.

Page: 424. Crepitus is a coarse, crackling sensation palpable over the skin surface. It occurs in subcutaneous emphysema when air escapes from the lung and enters the subcutaneous tissue, as after open thoracic injury or surgery.

  1. The nurse is auscultating the lungs of a patient who had been sleeping and notices short, popping, crackling sounds that stop after a few breaths. The nurse recognizes that these breath sounds are:

A) atelectatic crackles, and that they are not pathologic.
B) fine crackles, and that they may be a sign of pneumonia.
C) vesicular breath sounds.
D) fine wheezes.
A) atelectatic crackles, and that they are not pathologic.

Pages: 429-430. One type of adventitious sound, atelectatic crackles, is not pathologic. They are short, popping, crackling sounds that sound like fine crackles but do not last beyond a few breaths. When sections of alveoli are not fully aerated (as in people who are asleep or in the elderly), they deflate slightly and accumulate secretions. Crackles are heard when these sections are expanded by a few deep breaths. Atelectatic crackles are heard only in the periphery, usually in dependent portions of the lungs, and disappear after the first few breaths or after a cough.

  1. The nurse is assessing voice sounds during a respiratory assessment. Which of these findings indicates a normal assessment? Select all that apply.

A) Voice sounds are faint, muffled, and almost inaudible when the patient whispers “one, two, three” in a very soft voice.
B) As the patient says “ninety-nine” repeatedly, the examiner hears the words “ninety-nine” clearly.
C) When the patient speaks in a normal voice, the examiner can hear a sound but cannot distinguish exactly what is being said.
D) As the patient says a long “ee-ee-ee” sound, the examiner also hears a long “ee-ee-ee” sound.
E) As the patient says a long “ee-ee-ee” sound, the examiner hears a long “aaaaaa” sound.
A) Voice sounds are faint, muffled, and almost inaudible when the patient whispers “one, two, three” in a very soft voice.
C) When the patient speaks in a normal voice, the examiner can hear a sound but cannot distinguish exactly what is being said.
D) As the patient says a long “ee-ee-ee” sound, the examiner also hears a long “ee-ee-ee” sound.

Page: 446. As a patient says “ninety-nine” repeatedly, normally, the examiner hears sound but cannot distinguish what is being said. If a clear “ninety-nine” is auscultated, then it could indicate increased lung density, which enhances transmission of voice sounds. This is a measure of bronchophony. When a patient says a long “ee-ee-ee” sound, normally the examiner also hears a long “ee-ee-ee” sound through auscultation. This is a measure of egophony. If the examiner hears a long “aaaaaa” sound instead, this could indicate areas of consolidation or compression. With whispered pectoriloquy, as when a patient whispers a phrase such as “one-two-three,” the normal response when auscultating voice sounds is to hear sounds that are faint, muffled, and almost inaudible. If the examiners hears the whispered voice clearly, as if the patient is speaking through the stethoscope, then consolidation of the lung fields may exist.

  1. During an assessment of a 68-year-old man with a recent onset of right-sided weakness, the nurse hears a blowing, swishing sound with the bell of the stethoscope over the left carotid artery. This finding would indicate:

A) a valvular disorder.
B) blood flow turbulence.
C) fluid volume overload.
D) ventricular hypertrophy.
B) blood flow turbulence.

Page: 471. A bruit is a blowing, swishing sound indicating blood flow turbulence; normally none is present.

  1. During an assessment of a healthy adult, where would the nurse expect to palpate the apical impulse?

A) Third left intercostal space at the midclavicular line
B) Fourth left intercostal space at the sternal border
C) Fourth left intercostal space at the anterior axillary line
D) Fifth left intercostal space at the midclavicular line
D) Fifth left intercostal space at the midclavicular line

Pages: 473-474. The apical impulse should occupy only one intercostal space, the fourth or fifth, and it should be at or medial to the midclavicular line.

  1. The nurse is preparing to auscultate for heart sounds. Which technique is correct?

A) Listen to the sounds at the aortic, tricuspid, pulmonic, and mitral areas.
B) Listen by inching the stethoscope in a rough Z pattern, from the base of the heart across and down, then over to the apex.
C) Listen to the sounds only at the site where the apical pulse is felt to be the strongest.
D) Listen for all possible sounds at a time at each specified area.
B) Listen by inching the stethoscope in a rough Z pattern, from the base of the heart across and down, then over to the apex.

Pages: 475-476. Do not limit auscultation of breath sounds to only four locations. Sounds produced by the valves may be heard all over the precordium. Inch the stethoscope in a rough Z pattern from the base of the heart across and down, then over to the apex. Or, start at the apex and work your way up. See Figure 19-22. Listen selectively to one sound at a time.

  1. The nurse is assessing a patient’s apical impulse. Which of these statements is true regarding the apical impulse?

A) It is palpable in all adults.
B) It occurs with the onset of diastole.
C) Its location may be indicative of heart size.
D) It should normally be palpable in the anterior axillary line.
C) Its location may be indicative of heart size.

Page: 473 | Page: 492. The apical impulse is palpable in about 50% of adults. It is located in the fifth left intercostal space in the midclavicular line. Horizontal or downward displacement of the apical impulse may indicate an enlargement of the left ventricle.

  1. During an assessment of an older adult, the nurse should expect to notice which finding as a normal physiologic change associated with the aging process?

A) Hormonal changes causing vasodilation and a resulting drop in blood pressure
B) Progressive atrophy of the intramuscular calf veins, causing venous insufficiency
C) Peripheral blood vessels growing more rigid with age, producing a rise in systolic blood pressure
D) Narrowing of the inferior vena cava, causing low blood flow and increases in venous pressure resulting in varicosities
C) Peripheral blood vessels growing more rigid with age, producing a rise in systolic blood pressure

Pages: 504-505. Peripheral blood vessels grow more rigid with age, resulting in a rise in systolic blood pressure. Aging produces progressive enlargement of the intramuscular calf veins, not atrophy. The other options are not correct.

  1. During an assessment, the nurse uses the “profile sign” to detect:

A) pitting edema.
B) early clubbing.
C) symmetry of the fingers.
D) insufficient capillary refill.
B) early clubbing.

Page: 506. The nurse should use the profile sign (viewing the finger from the side) to detect early clubbing.

  1. When performing a peripheral vascular assessment on a patient, the nurse is unable to palpate the ulnar pulses. The patient’s skin is warm and capillary refill time is normal. The nurse should next:

A) check for the presence of claudication.
B) refer the individual for further evaluation.
C) consider this a normal finding and proceed with the peripheral vascular evaluation.
D) ask the patient if he or she has experienced any unusual cramping or tingling in the arm.
C) consider this a normal finding and proceed with the peripheral vascular evaluation.

Pages: 506-507. It is not usually necessary to palpate the ulnar pulses. The ulnar pulses are often not palpable in the normal person. The other responses are not correct.

  1. The nurse is attempting to assess the femoral pulse in an obese patient. Which of these actions would be most appropriate?

A) Have the patient assume a prone position.
B) Ask the patient to bend his or her knees to the side in a froglike position.
C) Press firmly against the bone with the patient in a semi-Fowler position.
D) Listen with a stethoscope for pulsations because it is very difficult to palpate the pulse in an obese person.
B) Regular “lub, dub” pattern

Pages: 510-511. To help expose the femoral area, particularly in obese people, the nurse should ask the person to bend his or her knees to the side in a froglike position.

  1. When using a Doppler ultrasonic stethoscope, the nurse recognizes arterial flow when which sound is heard?

A) Low humming sound
B) Regular “lub, dub” pattern
C) Swishing, whooshing sound
D) Steady, even, flowing sound
C) Swishing, whooshing sound

Pages: 515-516. When using the Doppler ultrasonic stethoscope, the pulse site is found when one hears a swishing, whooshing sound.

  1. The nurse is reviewing an assessment of a patient’s peripheral pulses and notices that the documentation states that the radial pulses are “2+.” The nurse recognizes that this reading indicates what type of pulse?

A) Bounding
B) Normal
C) Weak
D) Absent
B) Normal

Pages: 506-507. When documenting the force, or amplitude, of pulses, 3+ indicates an increased, full, or bounding pulse, 2+ indicates a normal pulse, 1+ indicates a weak pulse, and 0 indicates an absent pulse.

  1. The nurse is percussing the seventh right intercostal space at the midclavicular line over the liver. Which sound should the nurse expect to hear?

A) Dullness
B) Tympany
C) Resonance
D) Hyperresonance
A) Dullness

Page: 541. The liver is located in the right upper quadrant and would elicit a dull percussion note.

  1. Which structure is located in the left lower quadrant of the abdomen?

A) Liver
B) Duodenum
C) Gallbladder
D) Sigmoid colon
D) Sigmoid colon

Page: 530. The sigmoid colon is located in the left lower quadrant of the abdomen.

  1. The nurse suspects that a patient has a distended bladder. How should the nurse assess for this condition?

A) Percuss and palpate in the lumbar region.
B) Inspect and palpate in the epigastric region.
C) Auscultate and percuss in the inguinal region.
D) Percuss and palpate the midline area above the suprapubic bone.
D) Percuss and palpate the midline area above the suprapubic bone.

Pages: 539-540. Dull percussion sounds would be elicited over a distended bladder, and the hypogastric area would seem firm to palpation.

  1. While examining a patient, the nurse observes abdominal pulsations between the xiphoid and umbilicus. The nurse would suspect that these are:

A) pulsations of the renal arteries.
B) pulsations of the inferior vena cava.
C) normal abdominal aortic pulsations.
D) increased peristalsis from a bowel obstruction.
C) normal abdominal aortic pulsations.

Pages: 538-539. Normally, one may see the pulsations from the aorta beneath the skin in the epigastric area, particularly in thin persons with good muscle wall relaxation.

  1. A patient has hypoactive bowel sounds. The nurse knows that a potential cause of hypoactive bowel sounds is:

A) diarrhea.
B) peritonitis.
C) laxative use.
D) gastroenteritis.
B) peritonitis.

Page: 561. Diminished or absent bowel sounds signal decreased motility from inflammation as seen with peritonitis, with paralytic ileus after abdominal surgery, or with late bowel obstruction.

  1. The physician comments that a patient has abdominal borborygmi. The nurse knows that this term refers to:

A) a loud continuous hum.
B) a peritoneal friction rub.
C) hypoactive bowel sounds.
D) hyperactive bowel sounds.
D) hyperactive bowel sounds.

Pages: 539-540. Borborygmi is the term used for hyperperistalsis when the person actually feels his or her stomach growling.

  1. During an abdominal assessment, the nurse would consider which of these findings as normal?

A) The presence of a bruit in the femoral area
B) A tympanic percussion note in the umbilical region
C) A palpable spleen between the ninth and eleventh ribs in the left midaxillary line
D) A dull percussion note in the left upper quadrant at the midclavicular line
B) A tympanic percussion note in the umbilical region

Pages: 539-540. Tympany should predominate in all four quadrants of the abdomen because air in the intestines rises to the surface when the person is supine. Vascular bruits are not usually present. Normally the spleen is not palpable. Dullness would not be found in the area of lung resonance (left upper quadrant at the midclavicular line).

  1. During an abdominal assessment, the nurse is unable to hear bowel sounds in a patient’s abdomen. Before reporting this finding as “silent bowel sounds” the nurse should listen for at least:

A) 1 minute.
B) 5 minutes.
C) 10 minutes.
D) 2 minutes in each quadrant.
B) 5 minutes.

Pages: 539-540. Absent bowel sounds are rare. The nurse must listen for 5 minutes before deciding bowel sounds are completely absent.

  1. A patient is suspected of having inflammation of the gallbladder, or cholecystitis. The nurse should conduct which of these techniques to assess for this condition?

A) Obturator test
B) Test for Murphy’s sign
C) Assess for rebound tenderness
D) Iliopsoas muscle test
B) Test for Murphy’s sign

Page: 551. Normally, palpating the liver causes no pain. In a person with inflammation of the gallbladder, or cholecystitis, pain occurs as the descending liver pushes the inflamed gallbladder onto the examining hand during inspiration (Murphy’s test). The person feels sharp pain and abruptly stops inspiration midway.

  1. During an assessment the nurse notices that a patient’s umbilicus is enlarged and everted. It is midline, and there is no change in skin color. The nurse recognizes that the patient may have which condition?

A) Intra-abdominal bleeding
B) Constipation
C) Umbilical hernia
D) An abdominal tumor
C) Umbilical hernia

Page: 537. The umbilicus is normally midline and inverted, with no signs of discoloration. With an umbilical hernia, the mass is enlarged and everted. The other responses are incorrect.

  1. The nurse suspects that a patient has appendicitis. Which of these procedures are appropriate for use when assessing for appendicitis or a perforated appendix? Select all that apply.

A) Test for Murphy’s sign.
B) Test for Blumberg’s sign.
C) Test for shifting dullness.
D) Perform iliopsoas muscle test.
E) Test for fluid wave.
B) Test for Blumberg’s sign.
D) Perform iliopsoas muscle test.

Pages: 543-544 | Page: 551. Testing for Blumberg’s sign (rebound tenderness) and performing the iliopsoas muscle test should be used to assess for appendicitis. Murphy’s sign is used to assess for an inflamed gallbladder or cholecystitis. Testing for a fluid wave and shifting dullness is done to assess for ascites.

  1. When assessing muscle strength, the nurse observes that a patient has complete range of motion against gravity with full resistance. What Grade should the nurse record using a 0 to 5 point scale?

A) 2
B) 3
C) 4
D) 5
D) 5

Pages: 578-579. Complete range of motion against gravity is normal muscle strength and is recorded as Grade 5 muscle strength.

  1. The nurse is assessing the joints of a woman who has stated, “I have a long family history of arthritis, and my joints hurt.” The nurse suspects that she has osteoarthritis. Which of these are symptoms of osteoarthritis? Select all that apply.

A) Symmetric joint involvement
B) Asymmetric joint involvement
C) Pain with motion of affected joints
D) Affected joints are swollen with hard, bony protuberances
E) Affected joints may have heat, redness, and swelling
B) Asymmetric joint involvement
C) Pain with motion of affected joints
D) Affected joints are swollen with hard, bony protuberances

Page: 608. In osteoarthritis, asymmetric joint involvement commonly affects hands, knees, hips, and lumbar and cervical segments of the spine. Affected joints have stiffness, swelling with hard bony protuberances, pain with motion, and limitation of motion. The other options reflect signs of rheumatoid arthritis.

  1. During an assessment of an 80-year-old patient, the nurse notices the following: inability to identify vibrations at the ankle and to identify position of big toe, slower and more deliberate gait, and slightly impaired tactile sensation. All other neurologic findings are normal. The nurse should interpret that these findings indicate:

A) cranial nerve dysfunction.
B) lesion in the cerebral cortex.
C) normal changes due to aging.
D) demyelinization of nerves due to a lesion.
C) normal changes due to aging.

Page: 629. Some aging adults show a slower response to requests, especially for those calling for coordination of movements. The findings listed are normal in the absence of other significant abnormal findings. The other responses are incorrect.

  1. In obtaining a history on a 74-year-old patient the nurse notes that he drinks alcohol daily and that he has noticed a tremor in his hands that affects his ability to hold things. With this information, what should the nurse’s response be?

A) “Does your family know you are drinking every day?”
B) “Does the tremor change when you drink the alcohol?”
C) “We’ll do some tests to see what is causing the tremor.”
D) “You really shouldn’t drink so much alcohol; it may be causing your tremor.”
B) “Does the tremor change when you drink the alcohol?”

Page: 632. Intention tremor/ senile tremor is relieved by alcohol, although this is not a recommended treatment. The nurse should assess whether the person is abusing alcohol in an effort to relieve the tremor.

  1. During the neurologic assessment of a “healthy” 35-year-old patient, the nurse asks him to relax his muscles completely. The nurse then moves each extremity through full range of motion. Which of these results would the nurse expect to find?

A) Firm, rigid resistance to movement
B) Mild, even resistance to movement
C) Hypotonic muscles as a result of total relaxation
D) Slight pain with some directions of movement
B) Mild, even resistance to movement

Page: 637. Tone is the normal degree of tension (contraction) in voluntarily relaxed muscles. It shows a mild resistance to passive stretch. Normally, the nurse will notice a mild, even resistance to movement. The other responses are not correct.

  1. When the nurse asks a 68-year-old patient to stand with feet together and arms at his side with his eyes closed, he starts to sway and moves his feet farther apart. The nurse would document this finding as a(n):

A) ataxia.
B) lack of coordination.
C) negative Homans’ sign.
D) positive Romberg sign.
D) positive Romberg sign.

Page: 638. Abnormal findings for Romberg test include swaying, falling, and widening base of feet to avoid falling. Positive Romberg sign is loss of balance that is increased by closing of the eyes. Ataxia is uncoordinated or unsteady gait. Homans’ sign is used to test the legs for deep vein thrombosis.

  1. During the history of a 78-year-old man, his wife states that he occasionally has problems with short-term memory loss and confusion: “He can’t even remember how to button his shirt.” In doing the assessment of his sensory system, which action by the nurse is most appropriate?

A) The nurse would not do this part of the examination because results would not be valid.
B) The nurse would perform the tests, knowing that mental status does not affect sensory ability.
C) The nurse would proceed with the explanations of each test, making sure the wife understands.
D) Before testing, the nurse would assess the patient’s mental status and ability to follow directions at this time.
D) Before testing, the nurse would assess the patient’s mental status and ability to follow directions at this time.

The nurse should ensure validity of the sensory system testing by making sure the patient is alert, cooperative, comfortable, and has an adequate attention span. Otherwise, the nurse may obtain misleading and invalid results.

  1. In assessing a 70-year-old patient who has had a recent cerebrovascular accident, the nurse notices right-sided weakness. What might the nurse expect to find when testing his reflexes on the right side?

A) Lack of reflexes
B) Normal reflexes
C) Diminished reflexes
D) Hyperactive reflexes
D) Hyperactive reflexes

Hyperreflexia is the exaggerated reflex seen when the monosynaptic reflex arc is released from the influence of higher cortical levels. This occurs with upper motor neuron lesions (e.g., a cerebrovascular accident). The other responses are incorrect

  1. During the assessment of an 80-year-old patient, the nurse notices that his hands show tremors when he reaches for something and his head is always nodding. There is no associated rigidity with movement. Which of these statements is most accurate?

A) These are normal findings resulting from aging.
B) These could be related to hyperthyroidism.
C) These are the result of Parkinson disease.
D) This patient should be evaluated for a cerebellar lesion.
A) These are normal findings resulting from aging.

Page: 659. Senile tremors occasionally occur. These benign tremors include an intention tremor of the hands, head nodding (as if saying yes or no), and tongue protrusion. Tremors associated with Parkinson disease include rigidity, slowness, and weakness of voluntary movement. The other responses are incorrect.

  1. While the nurse is taking the history of a 68-year-old patient who sustained a head injury 3 days earlier, he tells the nurse that he is on a cruise ship and is 30 years old. The nurse knows that this finding is indicative of:

A) a great sense of humor.
B) uncooperative behavior.
C) inability to understand questions.
D) decreased level of consciousness.
D) decreased level of consciousness.

Pages: 660-661. A change in consciousness may be subtle. The nurse should notice any decreasing level of consciousness, disorientation, memory loss, uncooperative behavior, or even complacency in a previously combative person. The other responses are incorrect.

  1. The nurse is caring for a patient who has just had neurosurgery. To assess for increased intracranial pressure, what would the nurse include in the assessment?

A) Cranial nerves, motor function, and sensory function
B) Deep tendon reflexes, vital signs, and coordinated movements
C) Level of consciousness, motor function, pupillary response, and vital signs
D) Mental status, deep tendon reflexes, sensory function, and pupillary response
C) Level of consciousness, motor function, pupillary response, and vital signs

Pages: 660-661. Some hospitalized persons have head trauma or a neurologic deficit from a systemic disease process. These people must be monitored closely for any improvement or deterioration in neurologic status and for any indication of increasing intracranial pressure. The nurse should use an abbreviation of the neurologic examination in the following sequence: level of consciousness, motor function, pupillary response, and vital signs.

  1. During an assessment of a 22-year-old woman who has a head injury from a car accident 4 hours ago, the nurse notices the following change: pupils were equal, but now the right pupil is fully dilated and nonreactive, left pupil is 4 mm and reacts to light. What does finding this suggest?

A) Injury to the right eye
B) Increased intracranial pressure
C) Test was not performed accurately
D) Normal response after a head injury
B) Increased intracranial pressure

Pages: 662-663. In a brain-injured person, a sudden, unilateral, dilated, and nonreactive pupil is ominous. Cranial nerve III runs parallel to the brainstem. When increasing intracranial pressure pushes the brainstem down (uncal herniation), it puts pressure on cranial nerve III, causing pupil dilation. The other responses are incorrect.

  1. The nurse knows that determining whether a person is oriented to his or her surroundings will test the functioning of which of these structures?

A) Cerebrum
B) Cerebellum
C) Cranial nerves
D) Medulla oblongata
A) Cerebrum

Pages: 621-622 | Page: 660. The cerebral cortex is responsible for thought, memory, reasoning, sensation, and voluntary movement. The other options structures are not responsible for a person’s level of consciousness.

  1. The nurse is interviewing a patient who has a hearing impairment. What techniques would be most beneficial in communicating with this patient?

A) Determine the communication method he prefers.
B) Avoid using facial and hand gestures because most hearing-impaired people find this degrading.
C) Request a sign language interpreter before meeting with him to help facilitate the communication.
D) Speak loudly and with exaggerated facial movement when talking with him because this helps with lip reading.
A) Determine the communication method he prefers.

Pages: 40-41 The nurse should ask the deaf person the preferred way to communicate—by signing, lip reading, or writing. If the person prefers lip reading, then the nurse should be sure to face him or her squarely and have good lighting on the nurse’s face. The nurse should not exaggerate lip movements because this distorts words. Similarly, shouting distorts the reception of a hearing aid the person may wear. The nurse should speak slowly and should supplement his or her voice with appropriate hand gestures or pantomime.

  1. A female patient does not speak English well, and the nurse needs to choose an interpreter. Which of the following would be the most appropriate choice?

A) A trained interpreter
B) A male family member
C) A female family member
D) A volunteer college student from the foreign language studies department
A) A trained interpreter

Page: 46 whenever possible, the nurse should use a trained interpreter, preferably one who knows medical terminology. In general, an older, more mature interpreter is preferred to a younger, less experienced one, and the same gender is preferred when possible.

  1. In the majority culture of America, coughing, sweating, and diarrhea are symptoms of an illness. For some individuals of Mexican-American origin, however, these symptoms are a normal part of living. The nurse recognizes that this is true, probably because Mexican-Americans:

A) have less efficient immune systems and are often ill.
B) consider these symptoms a part of normal living, not symptoms of ill health.
C) come from Mexico and coughing is normal and healthy there.
D) are usually in a lower socioeconomic group and are more likely to be sick.
B) consider these symptoms a part of normal living, not symptoms of ill health.

Page: 27 The nurse needs to identify the meaning of health to the patient, remembering that concepts are derived, in part, from the way in which members of the cultural group define health.

  1. Among many Asians there is a belief in the yin/yang theory, rooted in the ancient Chinese philosophy of Tao. The nurse recognizes which statement that most accurately reflects “health” in an Asian with this belief?

A) A person is able to work and produce.
B) A person is happy, stable, and feels good.
C) All aspects of the person are in perfect balance.
D) A person is able to care for others and function socially.
C) All aspects of the person are in perfect balance.

Page: 21 Many Asians believe in the yin/yang theory, in which health is believed to exist when all aspects of the person are in perfect balance. The other statements do not describe this theory.

  1. If an American Indian has come to the clinic to seek help with regulating her diabetes, the nurse can expect that she:

A) will comply with the treatment prescribed.
B) has obviously given up her beliefs in naturalistic causes of disease.
C) may also be seeking the assistance of a shaman or medicine man.
D) will need extra help in dealing with her illness and may be experiencing a crisis of faith.
C) may also be seeking the assistance of a shaman or medicine man.

Page: 23 When self-treatment is unsuccessful, the individual may turn to the lay or folk healing systems, to spiritual or religious healing, or to scientific biomedicine. In addition to seeking help from a biomedical or scientific health care provider, patients may also seek help from folk or religious healers.

  1. An elderly Mexican-American woman with traditional beliefs has been admitted to an inpatient care unit. A culturally-sensitive nurse would:

A) contact the hospital administrator about the best course of action.
B) automatically get a curandero for her because it is not culturally appropriate for her to request one.
C) further assess the patient’s cultural beliefs and offer the patient assistance in contacting a curandero or priest if she desires.
D) ask the family what they would like to do because Mexican-Americans traditionally give control of decisions to their families.
C) further assess the patient’s cultural beliefs and offer the patient assistance in contacting a curandero or priest if she desires.

Pages: 22-23 In addition to seeking help from the biomedical/scientific health care provider, patients may also seek help from folk or religious healers. Some people, such as those of Mexican-American or American Indian origins, may believe that the cure is incomplete unless the body, mind, and spirit are also healed (although the division of the person into parts is a Western concept).

  1. The nurse is reviewing concepts of cultural aspects of pain. Which statement is true regarding pain?

A) All patients will behave the same way when in pain.
B) Just as patients vary in their perceptions of pain, so will they vary in their expressions of pain.
C) Cultural norms have very little to do with pain tolerance, because pain tolerance is always biologically determined.
D) A patient’s expression of pain is largely dependent on the amount of tissue injury associated with the pain.
B) Just as patients vary in their perceptions of pain, so will they vary in their expressions of pain.

Page: 25 In addition to expecting variations in pain perception and tolerance, the nurse should expect variations in the expression of pain. It is well known that individuals turn to their social environment for validation and comparison. The other statements are incorrect.

  1. The nurse recognizes that working with children with a different cultural perspective may be especially difficult because:

A) children have spiritual needs that are influenced by their stages of development.
B) children have spiritual needs that are direct reflections of what is occurring in their homes.
C) religious beliefs rarely affect the parents’ perceptions of the illness.
D) parents are often the decision makers, and they have no knowledge of their children’s spiritual needs.
A) children have spiritual needs that are influenced by their stages of development.

Page: 20. Illness during childhood may be an especially difficult clinical situation. Children, as well as adults, have spiritual needs that vary according to the child’s developmental level and the religious climate that exists in the family. The other statements are not correct.

  1. When planning a cultural assessment, the nurse should include which component?

A) Family history
B) Chief complaint
C) Medical history
D) Health-related beliefs
D) Health-related beliefs

Pages: 19-20. Health-related beliefs and practices are one component of a cultural assessment. The other items reflect other aspects of the patient’s history.

  1. When the nurse is evaluating the reliability of a patient’s responses, which of these statements would be correct? The patient:

A. has a history of drug abuse and therefore is not reliable.
B. provided consistent information and therefore is reliable.
C. smiled throughout interview and therefore is assumed reliable.
D. would not answer questions concerning stress and therefore is not reliable.
B. provided consistent information and therefore is reliable.

Page: 50. A reliable person always gives the same answers, even when questions are rephrased or are repeated later in the interview. The other statements are not correct.

  1. In recording the childhood illnesses of a patient who denies having had any, which note by the nurse would be most accurate?

A. Patient denies usual childhood illnesses.
B. Patient states he was a “very healthy” child.
C. Patient states sister had measles, but he didn’t.
D. Patient denies measles, mumps, rubella, chickenpox, pertussis, and strep throat.
D. Patient denies measles, mumps, rubella, chickenpox, pertussis, and strep throat.

Page: 51. Childhood illnesses include measles, mumps, rubella, chickenpox, pertussis, and strep throat. Avoid recording “usual childhood illnesses” because an illness common in the person’s childhood may be unusual today (e.g., measles).

  1. The nurse is performing a functional assessment on an 82-year-old patient who recently had a stroke. Which of these questions would be most important to ask?

A. “Do you wear glasses?”
B. “Are you able to dress yourself?”
C. “Do you have any thyroid problems?”
D. “How many times a day do you have a bowel movement?”
B. “Are you able to dress yourself?”

Page: 67. Functional assessment measures how a person manages day-to-day activities. For the older person, the meaning of health becomes those activities that they can or cannot do. The other responses do not relate to functional assessment.

  1. The nurse is conducting a developmental history on a 5-year-old child. Which questions are appropriate to ask the parents for this part of the assessment? Select all that apply.

A. “How much junk food does your child eat?”
B. “How many teeth has he lost, and when did he lose them?”
C. “Is he able to tie his shoelaces?”
D. “Does he take a children’s vitamin?”
E. “Can he tell time?”
F. “Does he have any food allergies?”
B. “How many teeth has he lost, and when did he lose them?”
C. “Is he able to tie his shoelaces?”
E. “Can he tell time?”

Page: 61. Questions about tooth loss, ability to tell time, and ability to tie shoelaces are appropriate questions for a developmental assessment. Questions about junk food intake and vitamins are part of a nutritional history. Questions about food allergies are not part of a developmental history.

  1. During an examination, the nurse can assess mental status by which activity?

A) Examining the patient’s electroencephalogram
B) Observing the patient as he or she performs an IQ test
C) Observing the patient and inferring health or dysfunction
D) Examining the patient’s response to a specific set of questions
C) Observing the patient and inferring health or dysfunction

Page: 71. Mental status cannot be scrutinized directly like the characteristics of skin or heart sounds. Its functioning is inferred through assessment of an individual’s behaviors, such as consciousness, language, mood and affect, and other aspects.

  1. The nurse is preparing to do a mental status examination. Which statement is true regarding the mental status examination?

A) A patient’s family is the best resource for information about the patient’s coping skills.
B) It is usually sufficient to gather mental status information during the health history interview.
C) It takes an enormous amount of extra time to integrate the mental status examination into the health history interview.
D) It is usually necessary to perform a complete mental status examination to get a good idea of the patient’s level of functioning.
B) It is usually sufficient to gather mental status information during the health history interview.

Page: 73. The full mental status examination is a systematic check of emotional and cognitive functioning. The steps described here, though, rarely need to be taken in their entirety. Usually, one can assess mental status through the context of the health history interview.

  1. The nurse is performing a mental status examination. Which statement is true regarding the assessment of mental status?

A) Mental status assessment diagnoses specific psychiatric disorders.
B) Mental disorders occur in response to everyday life stressors.
C) Mental status functioning is inferred through assessment of an individual’s behaviors.
D) Mental status can be assessed directly, just like other systems of the body (e.g., cardiac and breath sounds).
C) Mental status functioning is inferred through assessment of an individual’s behaviors.

Page: 71. Mental status functioning is inferred through assessment of an individual’s behaviors. It cannot be assessed directly like characteristics of the skin or heart sounds.

  1. When performing a physical assessment, the technique the nurse will always use first is:

A) palpation.
B) inspection.
C) percussion.
D) auscultation.
B) inspection.

Pages: 115-116. The skills requisite for the physical examination are inspection, palpation, percussion, and auscultation. The skills are performed one at a time and in this order (with the exception of the abdominal assessment, where auscultation takes place before palpation and percussion). The assessment of each body system begins with inspection. A focused inspection takes time and yields a surprising amount of information.

  1. The nurse is preparing to assess a patient’s abdomen by palpation. How should the nurse proceed?

A) Avoid palpation of reported “tender” areas because this may cause the patient pain.
B) Quickly palpate a tender area to avoid any discomfort that the patient may experience.
C) Begin the assessment with deep palpation, encouraging the patient to relax and take deep breaths.
D) Start with light palpation to detect surface characteristics and to accustom the patient to being touched.
D) Start with light palpation to detect surface characteristics and to accustom the patient to being touched.

Pages: 115-116. Light palpation is performed initially to detect any surface characteristics and to accustom the person to being touched. Tender areas should be palpated last, not first.

  1. The nurse would use bimanual palpation technique in which situation?

A) Palpating the thorax of an infant
B) Palpating the kidneys and uterus
C) Assessing pulsations and vibrations
D) Assessing the presence of tenderness and pain
B) Palpating the kidneys and uterus

Pages: 115-116. Bimanual palpation requires the use of both hands to envelop or capture certain body parts or organs such as the kidneys, uterus, or adnexa. The other situations are not appropriate for bimanual palpation.

  1. When performing a physical examination, safety must be considered to protect the examiner and the patient against the spread of infection. Which of these statements describes the most appropriate action the nurse should take when performing a physical examination?

A) There is no need to wash one’s hands after removing gloves, as long as the gloves are still intact.
B) Wash hands before and after every physical patient encounter.
C) Wash hands between the examination of each body system to prevent the spread of bacteria from one part of the body to another.
D) Wear gloves throughout the entire examination to demonstrate to the patient concern regarding the spread of infectious diseases.
B) Wash hands before and after every physical patient encounter.

Page: 120. The nurse should wash his or her hands before and after every physical patient encounter; after contact with blood, body fluids, secretions, and excretions; after contact with any equipment contaminated with body fluids; and after removing gloves. Hands should be washed after gloves have been removed, even if the gloves appear to be intact. Gloves should be worn when there is potential contact with any body fluids.

  1. When assessing the force, or strength, of a pulse, the nurse recalls that it:

A) is usually recorded on a 0- to 2-point scale.
B) demonstrates elasticity of the vessel wall.
C) is a reflection of the heart’s stroke volume.
D) reflects the blood volume in the arteries during diastole.
C) is a reflection of the heart’s stroke volume.

Page: 134. The heart pumps an amount of blood (the stroke volume) into the aorta. The force flares the arterial walls and generates a pressure wave, which is felt in the periphery as the pulse.

  1. The nurse needs to perform anthropometric measures of an 80-year-old man who is confined to a wheelchair. Which of the following is true in this situation?

A) Changes in fat distribution will affect the waist-to-hip ratio.
B) Height measurements may not be accurate because of changes in bone.
C) Declining muscle mass will affect the triceps skinfold measure.
D) Mid-arm circumference is difficult to obtain because of loss of skin elasticity.
B) Height measurements may not be accurate because of changes in bone.

Page: 191. Height measures may not be accurate in individuals confined to a bed or wheelchair or those over 60 years of age because of osteoporotic changes.

  1. The nurse is assessing for clubbing of the fingernails and would expect to find:

A) a nail base that is firm and slightly tender.
B) curved nails with a convex profile and ridges across the nail.
C) a nail base that feels spongy with an angle of the nail base of 150 degrees.
D) an angle of the nail base of 180 degrees or greater with a nail base that feels spongy.
D) an angle of the nail base of 180 degrees or greater with a nail base that feels spongy.

Pages: 217-218. The normal nail is firm at its base and has an angle of 160 degrees. In clubbing, the angle straightens to 180 degrees or greater and the nail base feels spongy.

  1. A patient has been admitted for severe psoriasis. The nurse can expect to see what finding in the patient’s fingernails?

A) Splinter hemorrhages
B) Paronychia
C) Pitting
D) Beau lines
C) Pitting

Pages: 248-250. Pitting nails are characterized by sharply defined pitting and crumbling of the nails with distal detachment, and they are associated with psoriasis. See Table 12-13 for descriptions of the other terms.

  1. The nurse suspects that a patient has hyperthyroidism and laboratory data indicate that the patient’s thyroxine and tri-iodothyronine hormone levels are elevated. Which of these findings would the nurse most likely find on examination?

A) Tachycardia
B) Constipation
C) Rapid dyspnea
D) Atrophied nodular thyroid
A) Tachycardia

Thyroxine and tri-iodothyronine are thyroid hormones that stimulate the rate of cellular metabolism, resulting in tachycardia. With an enlarged thyroid as in hyperthyroidism, the nurse might expect to find diffuse enlargement (goiter) or a nodular lump, but not an atrophied gland. Dyspnea and constipation are not findings associated with hyperthyroidism.

  1. During an examination, the nurse knows that Paget’s disease would be indicated by which of these assessment findings?

A) Positive Macewen sign
B) Premature closure of the sagittal suture
C) Headache, vertigo, tinnitus, and deafness
D) Elongated head with heavy eyebrow ridge
C) Headache, vertigo, tinnitus, and deafness

Paget’s disease occurs more often in males and is characterized by bowed long bones, sudden fractures, and enlarging skull bones that press on cranial nerves causing symptoms of headache, vertigo, tinnitus, and progressive deafness.

  1. A woman comes to the clinic and states, “I’ve been sick for so long! My eyes have gotten so puffy, and my eyebrows and hair have become coarse and dry.” The nurse will assess for other signs and symptoms of:

A) cachexia.
B) Parkinson’s syndrome.
C) myxedema.
D) scleroderma.
C) myxedema.

Pages: 276-277. Myxedema (hypothyroidism) is a deficiency of thyroid hormone that, when severe, causes a nonpitting edema or myxedema. The patient will have a puffy edematous face especially around eyes (periorbital edema), coarse facial features, dry skin, and dry, coarse hair and eyebrows. See Table 13-4, Abnormal Facial Appearances with Chronic Illnesses, for descriptions of the other responses.

  1. During an assessment of an infant, the nurse notes that the fontanels are depressed and sunken. The nurse suspects which condition?

A) Rickets
B) Dehydration
C) Mental retardation
D) Increased intracranial pressure
B) Dehydration

Pages: 265-266. Depressed and sunken fontanels occur with dehydration or malnutrition. Mental retardation and rickets have no effect on fontanels. Increased intracranial pressure would cause tense or bulging, and possibly pulsating fontanels.

  1. A mother asks when her newborn infant’s eyesight will be developed. The nurse should reply:

A) “Vision is not totally developed until 2 years of age.”
B) “Infants develop the ability to focus on an object at around 8 months.”
C) “By about 3 months, infants develop more coordinated eye movements and can fixate on an object.”
D) “Most infants have uncoordinated eye movements for the first year of life.”
C) “By about 3 months, infants develop more coordinated eye movements and can fixate on an object.”

Page: 284. Eye movements may be poorly coordinated at birth, but by 3 to 4 months of age, the infant should establish binocularity and should be able to fixate on a single image with both eyes simultaneously.

  1. The nurse notices the presence of periorbital edema when performing an eye assessment on a 70-year-old patient. The nurse should:

A) check for the presence of exophthalmos.
B) suspect that the patient has hyperthyroidism.
C) ask the patient if he or she has a history of heart failure.
D) assess for blepharitis because this is often associated with periorbital edema.
C) ask the patient if he or she has a history of heart failure.

Page: 312. Periorbital edema occurs with local infections, crying, and systemic conditions such as heart failure, renal failure, allergy, and hypothyroidism. Periorbital edema is not associated with blepharitis.

  1. A patient comes to the emergency department after a boxing match, and his left eye is swollen almost shut. He has bruises on his face and neck. He says he is worried because he “can’t see well” from his left eye. The physician suspects retinal damage. The nurse recognizes that signs of retinal detachment include:

A) loss of central vision.
B) shadow or diminished vision in one quadrant or one half of the visual field.
C) loss of peripheral vision.
D) sudden loss of pupillary constriction and accommodation.
B) shadow or diminished vision in one quadrant or one half of the visual field.

Page: 316. With retinal detachment, the person has shadows or diminished vision in one quadrant or one half of the visual field. The other responses are not signs of retinal detachment.

  1. A 68-year-old woman is in the eye clinic for a checkup. She tells the nurse that she has been having trouble with reading the paper, sewing, and even seeing the faces of her grandchildren. On examination, the nurse notes that she has some loss of central vision but her peripheral vision is normal. These findings suggest that:

A) she may have macular degeneration.
B) her vision is normal for someone her age.
C) she has the beginning stages of cataract formation.
D) she has increased intraocular pressure or glaucoma.
A) she may have macular degeneration.

Page: 285. Macular degeneration is the most common cause of blindness. It is characterized by loss of central vision. Cataracts would show lens opacity. Chronic open-angle glaucoma, the most common type of glaucoma, involves a gradual loss of peripheral vision.

  1. An ophthalmic examination reveals papilledema. The nurse is aware that this finding indicates:

A) retinal detachment.
B) diabetic retinopathy.
C) acute-angle glaucoma.
D) increased intracranial pressure.
D) increased intracranial pressure.

Pages: 319-320. Papilledema, or choked disk, is a serious sign of increased intracranial pressure, which is caused by a space-occupying mass such as a brain tumor or hematoma. This pressure causes venous stasis in the globe, showing redness, congestion, and elevation of the optic disc, blurred margins, hemorrhages, and absent venous pulsations. Papilledema is not associated with the conditions in the other responses.

  1. During an examination, a patient states that she was diagnosed with open-angle glaucoma 2 years ago. The nurse assesses for characteristics of open-angle glaucoma. Which of these are characteristics of open-angle glaucoma? Select all that apply.

A) The patient may experience sensitivity to light, nausea, and halos around lights.
B) The patient experiences tunnel vision in late stages.
C) Immediate treatment is needed.
D) Vision loss begins with peripheral vision.
E) It causes sudden attacks of increased pressure that cause blurred vision.
F) There are virtually no symptoms.
B) The patient experiences tunnel vision in late stages.
D) Vision loss begins with peripheral vision.
F) There are virtually no symptoms.

Pages: 308-309. Open-angle glaucoma is the most common type of glaucoma; there are virtually no symptoms. Vision loss begins with the peripheral vision, which often goes unnoticed because individuals learn to compensate intuitively by turning their heads. The other characteristics are those of closed-angle glaucoma.

  1. The nurse is testing the hearing of a 78-year-old man and keeps in mind the changes in hearing that occur with aging include which of the following? Select all that apply.

A) Hearing loss related to aging begins in the mid 40s.
B) The progression is slow.
C) The aging person has low-frequency tone loss.
D) The aging person may find it harder to hear consonants than vowels.
E) Sounds may be garbled and difficult to localize.
F) Hearing loss reflects nerve degeneration of the middle ear.
B) The progression is slow.
D) The aging person may find it harder to hear consonants than vowels.
E) Sounds may be garbled and difficult to localize.

Page: 326. Presbycusis is a type of hearing loss that occurs with aging and is found in 60% of those older than 65 years. It is a gradual sensorineural loss caused by nerve degeneration in the inner ear or auditory nerve, and it slowly progresses after age 50. The person first notices a high-frequency tone loss; it is harder to hear consonants (high-pitched components of speech) than vowels. This makes words sound garbled. The ability to localize sound is impaired also.

  1. During an assessment, the nurse knows that expected assessment findings in the normal adult lung include the presence of:

A) adventitious sounds and limited chest expansion.
B) increased tactile fremitus and dull percussion tones.
C) muffled voice sounds and symmetrical tactile fremitus.
D) absent voice sounds and hyperresonant percussion tones.
C) muffled voice sounds and symmetrical tactile fremitus.

Pages: 429-430. Normal lung findings include symmetric chest expansion, resonant percussion tones, vesicular breath sounds over the peripheral lung fields, muffled voice sounds, and no adventitious sounds.

  1. When assessing tactile fremitus, the nurse recalls that it is normal to feel tactile fremitus most intensely over which location?

A) Between the scapulae
B) Third intercostal space, MCL
C) Fifth intercostal space, MAL
D) Over the lower lobes, posterior side
A) Between the scapulae

Page: 424. Normally, fremitus is most prominent between the scapulae and around the sternum. These are sites where the major bronchi are closest to the chest wall. Fremitus normally decreases as one progress down the chest because more tissue impedes sound transmission.

  1. The nurse is reviewing the technique of palpating for tactile fremitus with a new graduate. Which statement by the graduate nurse reflects a correct understanding of tactile fremitus? “Tactile fremitus:

A) is caused by moisture in the alveoli.”
B) indicates that there is air in the subcutaneous tissues.”
C) is caused by sounds generated from the larynx.”
D) reflects the blood flow through the pulmonary arteries.”
C) is caused by sounds generated from the larynx.”

Pages: 422-423. Fremitus is a palpable vibration. Sounds generated from the larynx are transmitted through patent bronchi and the lung parenchyma to the chest wall where they are felt as vibrations. Crepitus is the term for air in the subcutaneous tissues.

  1. When auscultating the lungs of an adult patient, the nurse notes that over the posterior lower lobes low-pitched, soft breath sounds are heard, with inspiration being longer than expiration. The nurse interprets that these are:

A) sounds normally auscultated over the trachea.
B) bronchial breath sounds and are normal in that location.
C) vesicular breath sounds and are normal in that location.
D) bronchovesicular breath sounds and are normal in that location.
C) vesicular breath sounds and are normal in that location.

Pages: 428-429. Vesicular breath sounds are low-pitched, soft sounds with inspiration being longer than expiration. These breath sounds are expected over peripheral lung fields where air flows through smaller bronchioles and alveoli.

  1. The nurse is percussing over the lungs of a patient with pneumonia. The nurse knows that percussion over an area of atelectasis in the lungs would reveal:

A) dullness.
B) tympany.
C) resonance.
D) hyperresonance.
A) dullness.

Pages: 424-425. A dull percussion note signals an abnormal density in the lungs, as with pneumonia, pleural effusion, atelectasis, or tumor.

  1. The nurse knows that auscultation of fine crackles would most likely be noticed in:

A) a healthy 5-year-old child.
B) a pregnant woman.
C) the immediate newborn period.
D) association with a pneumothorax.
C) the immediate newborn period.

Pages: 436-437. Fine crackles are commonly heard in the immediate newborn period as a result of the opening of the airways and clearing of fluid. Persistent fine crackles would be noticed with pneumonia, bronchiolitis, or atelectasis.

  1. During auscultation of the lungs of an adult patient, the nurse notices the presence of bronchophony. The nurse should assess for signs of which condition?

A) Airway obstruction
B) Emphysema
C) Pulmonary consolidation
D) Asthma
C) Pulmonary consolidation

Page: 446. Pathologic conditions that increase lung density, such as pulmonary consolidation, will enhance transmission of voice sounds, such as bronchophony. See Table 18-7.

  1. The nurse is listening to the breath sounds of a patient with severe asthma. Air passing through narrowed bronchioles would produce which of these adventitious sounds?

A) Wheezes
B) Bronchial sounds
C) Bronchophony
D) Whispered pectoriloquy
A) Wheezes

Page: 445. Wheezes are caused by air squeezed or compressed through passageways narrowed almost to closure by collapsing, swelling, secretions, or tumors, such as with acute asthma or chronic emphysema.

  1. An adult patient with a history of allergies comes to the clinic complaining of wheezing and difficulty in breathing when working in his yard. The assessment findings include tachypnea, use of accessory neck muscles, prolonged expiration, intercostal retractions, decreased breath sounds, and expiratory wheezes. The nurse interprets that these assessment findings are consistent with:

A) asthma.
B) atelectasis.
C) lobar pneumonia.
D) heart failure.
A) asthma.

Page: 451. Asthma is allergic hypersensitivity to certain inhaled particles that produces inflammation and a reaction of bronchospasm, which increases airway resistance, especially during expiration. Increased respiratory rate, use of accessory muscles, retraction of intercostal muscles, prolonged expiration, decreased breath sounds, and expiratory wheezing are all characteristic of asthma. See Table 18-8 for descriptions of the other conditions.

  1. During palpation of the anterior chest wall, the nurse notices a coarse, crackling sensation over the skin surface. On the basis of these findings, the nurse suspects:

A) tactile fremitus.
B) crepitus.
C) friction rub.
D) adventitious sounds.
B) crepitus.

Page: 424. Crepitus is a coarse, crackling sensation palpable over the skin surface. It occurs in subcutaneous emphysema when air escapes from the lung and enters the subcutaneous tissue, as after open thoracic injury or surgery.

  1. The nurse is auscultating the lungs of a patient who had been sleeping and notices short, popping, crackling sounds that stop after a few breaths. The nurse recognizes that these breath sounds are:

A) atelectatic crackles, and that they are not pathologic.
B) fine crackles, and that they may be a sign of pneumonia.
C) vesicular breath sounds.
D) fine wheezes.
A) atelectatic crackles, and that they are not pathologic.

Pages: 429-430. One type of adventitious sound, atelectatic crackles, is not pathologic. They are short, popping, crackling sounds that sound like fine crackles but do not last beyond a few breaths. When sections of alveoli are not fully aerated (as in people who are asleep or in the elderly), they deflate slightly and accumulate secretions. Crackles are heard when these sections are expanded by a few deep breaths. Atelectatic crackles are heard only in the periphery, usually in dependent portions of the lungs, and disappear after the first few breaths or after a cough.

  1. The nurse is assessing voice sounds during a respiratory assessment. Which of these findings indicates a normal assessment? Select all that apply.

A) Voice sounds are faint, muffled, and almost inaudible when the patient whispers “one, two, three” in a very soft voice.
B) As the patient says “ninety-nine” repeatedly, the examiner hears the words “ninety-nine” clearly.
C) When the patient speaks in a normal voice, the examiner can hear a sound but cannot distinguish exactly what is being said.
D) As the patient says a long “ee-ee-ee” sound, the examiner also hears a long “ee-ee-ee” sound.
E) As the patient says a long “ee-ee-ee” sound, the examiner hears a long “aaaaaa” sound.
A) Voice sounds are faint, muffled, and almost inaudible when the patient whispers “one, two, three” in a very soft voice.
C) When the patient speaks in a normal voice, the examiner can hear a sound but cannot distinguish exactly what is being said.
D) As the patient says a long “ee-ee-ee” sound, the examiner also hears a long “ee-ee-ee” sound.

Page: 446. As a patient says “ninety-nine” repeatedly, normally, the examiner hears sound but cannot distinguish what is being said. If a clear “ninety-nine” is auscultated, then it could indicate increased lung density, which enhances transmission of voice sounds. This is a measure of bronchophony. When a patient says a long “ee-ee-ee” sound, normally the examiner also hears a long “ee-ee-ee” sound through auscultation. This is a measure of egophony. If the examiner hears a long “aaaaaa” sound instead, this could indicate areas of consolidation or compression. With whispered pectoriloquy, as when a patient whispers a phrase such as “one-two-three,” the normal response when auscultating voice sounds is to hear sounds that are faint, muffled, and almost inaudible. If the examiners hears the whispered voice clearly, as if the patient is speaking through the stethoscope, then consolidation of the lung fields may exist.

  1. During an assessment of a 68-year-old man with a recent onset of right-sided weakness, the nurse hears a blowing, swishing sound with the bell of the stethoscope over the left carotid artery. This finding would indicate:

A) a valvular disorder.
B) blood flow turbulence.
C) fluid volume overload.
D) ventricular hypertrophy.
B) blood flow turbulence.

Page: 471. A bruit is a blowing, swishing sound indicating blood flow turbulence; normally none is present.

  1. The nurse is assessing a patient’s apical impulse. Which of these statements is true regarding the apical impulse?

A) It is palpable in all adults.
B) It occurs with the onset of diastole.
C) Its location may be indicative of heart size.
D) It should normally be palpable in the anterior axillary line.
C) Its location may be indicative of heart size.

Page: 473 | Page: 492. The apical impulse is palpable in about 50% of adults. It is located in the fifth left intercostal space in the midclavicular line. Horizontal or downward displacement of the apical impulse may indicate an enlargement of the left ventricle.

  1. During an assessment, the nurse uses the “profile sign” to detect:

A) pitting edema.
B) early clubbing.
C) symmetry of the fingers.
D) insufficient capillary refill.
B) early clubbing.

Page: 506. The nurse should use the profile sign (viewing the finger from the side) to detect early clubbing.

  1. The nurse is attempting to assess the femoral pulse in an obese patient. Which of these actions would be most appropriate?

A) Have the patient assume a prone position.
B) Ask the patient to bend his or her knees to the side in a froglike position.
C) Press firmly against the bone with the patient in a semi-Fowler position.
D) Listen with a stethoscope for pulsations because it is very difficult to palpate the pulse in an obese person.
B) Regular “lub, dub” pattern

Pages: 510-511. To help expose the femoral area, particularly in obese people, the nurse should ask the person to bend his or her knees to the side in a froglike position.

  1. When using a Doppler ultrasonic stethoscope, the nurse recognizes arterial flow when which sound is heard?

A) Low humming sound
B) Regular “lub, dub” pattern
C) Swishing, whooshing sound
D) Steady, even, flowing sound
C) Swishing, whooshing sound

Pages: 515-516. When using the Doppler ultrasonic stethoscope, the pulse site is found when one hears a swishing, whooshing sound.

  1. The nurse is reviewing an assessment of a patient’s peripheral pulses and notices that the documentation states that the radial pulses are “2+.” The nurse recognizes that this reading indicates what type of pulse?

A) Bounding
B) Normal
C) Weak
D) Absent
B) Normal

Pages: 506-507. When documenting the force, or amplitude, of pulses, 3+ indicates an increased, full, or bounding pulse, 2+ indicates a normal pulse, 1+ indicates a weak pulse, and 0 indicates an absent pulse.

  1. The nurse is percussing the seventh right intercostal space at the midclavicular line over the liver. Which sound should the nurse expect to hear?

A) Dullness
B) Tympany
C) Resonance
D) Hyperresonance
A) Dullness

Page: 541. The liver is located in the right upper quadrant and would elicit a dull percussion note.

  1. Which structure is located in the left lower quadrant of the abdomen?

A) Liver
B) Duodenum
C) Gallbladder
D) Sigmoid colon
D) Sigmoid colon

Page: 530. The sigmoid colon is located in the left lower quadrant of the abdomen.

  1. While examining a patient, the nurse observes abdominal pulsations between the xiphoid and umbilicus. The nurse would suspect that these are:

A) pulsations of the renal arteries.
B) pulsations of the inferior vena cava.
C) normal abdominal aortic pulsations.
D) increased peristalsis from a bowel obstruction.
C) normal abdominal aortic pulsations.

Pages: 538-539. Normally, one may see the pulsations from the aorta beneath the skin in the epigastric area, particularly in thin persons with good muscle wall relaxation.

  1. A patient has hypoactive bowel sounds. The nurse knows that a potential cause of hypoactive bowel sounds is:

A) diarrhea.
B) peritonitis.
C) laxative use.
D) gastroenteritis.
B) peritonitis.

Page: 561. Diminished or absent bowel sounds signal decreased motility from inflammation as seen with peritonitis, with paralytic ileus after abdominal surgery, or with late bowel obstruction.

  1. During an abdominal assessment, the nurse would consider which of these findings as normal?

A) The presence of a bruit in the femoral area
B) A tympanic percussion note in the umbilical region
C) A palpable spleen between the ninth and eleventh ribs in the left midaxillary line
D) A dull percussion note in the left upper quadrant at the midclavicular line
B) A tympanic percussion note in the umbilical region

Pages: 539-540. Tympany should predominate in all four quadrants of the abdomen because air in the intestines rises to the surface when the person is supine. Vascular bruits are not usually present. Normally the spleen is not palpable. Dullness would not be found in the area of lung resonance (left upper quadrant at the midclavicular line).

  1. A patient is suspected of having inflammation of the gallbladder, or cholecystitis. The nurse should conduct which of these techniques to assess for this condition?

A) Obturator test
B) Test for Murphy’s sign
C) Assess for rebound tenderness
D) Iliopsoas muscle test
B) Test for Murphy’s sign

Page: 551. Normally, palpating the liver causes no pain. In a person with inflammation of the gallbladder, or cholecystitis, pain occurs as the descending liver pushes the inflamed gallbladder onto the examining hand during inspiration (Murphy’s test). The person feels sharp pain and abruptly stops inspiration midway.

  1. During an assessment the nurse notices that a patient’s umbilicus is enlarged and everted. It is midline, and there is no change in skin color. The nurse recognizes that the patient may have which condition?

A) Intra-abdominal bleeding
B) Constipation
C) Umbilical hernia
D) An abdominal tumor
C) Umbilical hernia

Page: 537. The umbilicus is normally midline and inverted, with no signs of discoloration. With an umbilical hernia, the mass is enlarged and everted. The other responses are incorrect.

  1. The nurse suspects that a patient has appendicitis. Which of these procedures are appropriate for use when assessing for appendicitis or a perforated appendix? Select all that apply.

A) Test for Murphy’s sign.
B) Test for Blumberg’s sign.
C) Test for shifting dullness.
D) Perform iliopsoas muscle test.
E) Test for fluid wave.
B) Test for Blumberg’s sign.
D) Perform iliopsoas muscle test.

Pages: 543-544 | Page: 551. Testing for Blumberg’s sign (rebound tenderness) and performing the iliopsoas muscle test should be used to assess for appendicitis. Murphy’s sign is used to assess for an inflamed gallbladder or cholecystitis. Testing for a fluid wave and shifting dullness is done to assess for ascites.

  1. During the neurologic assessment of a “healthy” 35-year-old patient, the nurse asks him to relax his muscles completely. The nurse then moves each extremity through full range of motion. Which of these results would the nurse expect to find?

A) Firm, rigid resistance to movement
B) Mild, even resistance to movement
C) Hypotonic muscles as a result of total relaxation
D) Slight pain with some directions of movement
B) Mild, even resistance to movement

Page: 637. Tone is the normal degree of tension (contraction) in voluntarily relaxed muscles. It shows a mild resistance to passive stretch. Normally, the nurse will notice a mild, even resistance to movement. The other responses are not correct.

  1. During an assessment of a 22-year-old woman who has a head injury from a car accident 4 hours ago, the nurse notices the following change: pupils were equal, but now the right pupil is fully dilated and nonreactive, left pupil is 4 mm and reacts to light. What does finding this suggest?

A) Injury to the right eye
B) Increased intracranial pressure
C) Test was not performed accurately
D) Normal response after a head injury
B) Increased intracranial pressure

Pages: 662-663. In a brain-injured person, a sudden, unilateral, dilated, and nonreactive pupil is ominous. Cranial nerve III runs parallel to the brainstem. When increasing intracranial pressure pushes the brainstem down (uncal herniation), it puts pressure on cranial nerve III, causing pupil dilation. The other responses are incorrect.

  1. The nurse knows that determining whether a person is oriented to his or her surroundings will test the functioning of which of these structures?

A) Cerebrum
B) Cerebellum
C) Cranial nerves
D) Medulla oblongata
A) Cerebrum

Pages: 621-622 | Page: 660. The cerebral cortex is responsible for thought, memory, reasoning, sensation, and voluntary movement. The other options structures are not responsible for a person’s level of consciousness.

The nurse is performing a thoracic assessment on a client with chronic asthma and hyperinflation of the lungs. Which finding should be expected for this client?
Barrel chest

The nurse is assessing bowel sounds for a hospitalized client. The nurse has heard bowel sounds in the right upper quadrant. What action should the nurse take next?
Note the character and frequency of bowel sounds

During inspection of a client’s mouth and pharynx, the nurse places a tongue blade on the back of the tongue which causes the client to gag. After removing the tongue blade, what action should the nurse take?
Document an intact gag reflex.

When teaching a client how to perform a monthly breast self-assessment, the nurse should tell the client that it is most important to assess which part of the breast more closely for changes?
Upper outer quadrant.

The nurse is assessing a postmenopausal client who has a BMI of 32. The client has a chest measurement of 42 inches, waist measurement of 45 inches, and hip measurement of 50 inches. What important message should the nurse explain to the client to promote health promotion?
A waist circumference is greater than 35 inches in women puts you at higher risk for type 2 diabetes and heart disease.”

The nurse performs a physical assessment on an older female client. Which change from the prior exam may be an indication of osteoporosis?
Height reduction of 1.5 inches.

While conducting an interview to obtain a health history, the nurse notices that the client pauses frequently and looks at the nurse expectantly. Which response is best for the nurse to provide?
Sit quietly to allow the client to respond comfortably.

A client is in the clinical for a yearly physical examination. Which action should the nurse take when preparing to examine the client’s abdomen?
Ask the client to urinate before beginning the examination.

Which respiratory condition should the nurse document after measuring a respiratory rate of 8 breaths/minute?
Bradypnea.

Which procedure should the nurse use to assessfor a pulse deficit?
Measure the apical pulse and compare it to the peripheral pulse.

*A pulse deficit is a palpable difference between the apical pulse at the point of maximal impulse and the radial pulse palpated at the wrist.

A client has been diagnosed with bilateral lower lobe atelectasis. What percussion sound should the nurse expect to hear when percussing over the client’s lower lobes?
Dull, thud-like.

A client is being assessed upon admission to the medical-surgical unit. The nurse is preparing to complete a head-to-toe assessment and will begin at the head of the client. Which technique should the nurse use to begin the assessment?
Inspect the hair and skin.

The nurse is assessing a healthy young adult during an annual physical examination. Which assessment technique should the nurse implement when palpating the abdominal aorta?
Deep palpation above and to the left of the umbilicus.

The nurse is conducting a family history as part of the assessment interview. Which action should the nurse take to ensure that sufficient information about the client’s blood relatives is obtained?
Document at least 3 generations of the client’s family medical history.

The nurse is testing the client’s shoulders for range of motion. What should the nurse document to record normal internal rotation?
Range of 90 degrees when the hands are placed at the small of the back.

A client presents with a rash along the occipital area of the hairline and reports intense itching. How should the nurse begin the objective part of the examination?
Inspect the scalp looking for nits.

The nurse is assessing a client’s range of motion as the client bends the right knee up to the chest while keeping the left leg straight, but is unable to keep the left thigh on the table. The assessment is repeated for the left knee, and the client is unable to keep the right thigh on the table. How should the nurse document this finding?
A flexion deformity referred to as a positive Thomas test.

During a skin asssessment, the nurse notes, round and discrete lesions that are dark red in color and will not blanch. The lesions range from 1 to 3 mm in size. What is the first question the nurse should ask the client?
Have you notice any irregular bleeding

A client with progressive hearing loss appears distressed when the registered nurse (RN) asks open-ended questions about the client’s health history. Which forms of communication should the RN use?
Face the client so the client can see the RN’s mouth.
Check if the client’s hearing aides are working properly.
Reduce environmental noise surrounding the client.

A client states that she had a mastectomy of her left breast last year and now experiences lymphedema. What should the nurse expect to find when examining the client?
Swelling of the left arm and non-pitting edema.

A client has just returned from the recovery room and asks to get out of bed to go to the bathroom. The nurse decides to obtain orthostatic vital signs first. How will the nurse position the client to begin this procedure?
Lying.

A postmenopausal female client is undergoing a routine physical examination. She has reported nothing out of the ordinary. When performing the examination of the genitourinary system, the nurse finds an irregularly enlarged uterus with firm, mobile, painless nodules in the uterine wall. How should the nurse explain this finding to the client?
You have benign fibroid tumors, a common occurrence in women your age.

A client is reporting chest pain. What statement made by the client, helps the nurse to understand this client has a naturalistic belief in the cause of illness?
“My life is really out of balance.”

The nurse is preparing to assess the hearing of a client with a history of prolonged exposure to occupational noise. Which hearing test provides the most reliable assessment of hearing status?
Audiometry.

The nurse is performing a routine physical examination on an adult client. When gathering a health history, which question is included in the CAGE questionnaire?
Have you ever felt guilty about your drinking?

*CAGE is the acronym for Cut down, Annoyed, Guilty, and Eye-opener. Nurse can use it to assess for possible alcohol abuse.

The nurse is examining the hip joint of a client who reports hip pain. Which other assessment is most helpful in determining the cause of the client’s pain?
Knee joint evaluation.

The nurse performs a series of cranial nerve tests on a client with a head injury. Which test should the nurse use to assess damage to the first cranial nerve?
Occlude one nostril and have the client identify various odors.

The client reports to the nurse a recent exposure to the mumps. Which assessment finding suggests the client has contracted the mumps?
Swelling anterior to the ear lobe on one side of the face

A nurse is working in a healthcare facility that serves a diverse population. What action(s) by the nurse will allow the nurse to empathize with and understand this population? (Select all that apply.)
Be open to people who are different.
Have a curiosity about people.
Become culturally competent.

Which findings can the nurse determine by palpating a client’s skin? (Select all that apply.)
Diaphoresis.
Scaling.

Which question should the nurse ask in order to test a client’s remote memory?
What is your date of birth?

While assessing level of consciousness, the nurse finds that a client localizes to pain, is confused during conversation, and opens the eyes to sound. How should the nurse document the Glasgow score of this client?
12.

The Glasgow Coma Scale is used to establish baseline data based on eye opening, motor response, and verbal response. The lowest possible score is 3 and thehighest is 15. This client’s Glasgow Coma Scale (GCS) score is 12: Opening eyes to sound is a score of 3, localizing to pain is a 5, and confusion during a conversation is a 4 (3 + 5 + 4 = 12).

A client is in the clinic and is reporting lower abdominal pain and constipation. Which information is of greatest concern to the nurse when obtaining the health history from this client?
Family history of colon cancer on mother’s side.

An adult client is in the clinic for a regular physical examination. The nurse is assessing the client’s hydration status by pinching then releasing the client’s skin. Which finding is indicative of good hydration status?
The skin immediately returns to normal position.

A client comes to the clinic with a report of fever and a recent exposure to someone who was diagnosed with meningitis. Which nursing assessment should be completed during the initial examination of this client?
Level of consciousness.

While palpating a client’s breasts, the nurse detects a nontender, solitary, round lobular mass that is solid and firm and slides easily through the breast tissue . The findings of this breast exam are consistent with which condition?
Fibroadenoma.

The client is experiencing severe pruritus and small papules and burrows on areas over one hand and the inner thighs. Which assessment data best explains the condition the client is experiencing?
The client works in a daycare setting that has had a scabies outbreak.

When assessing facial nerve function of a 96-year-old, the nurse asks the client to smile in an exaggerated manner. Which finding is most important for the nurse to further asses?
Only one side of the mouth moves when smiling.

When performing range of motion exercises on the joints of an older adult client, the nurse notes that joint range is greater with passive ranging than with active ranging. A goniometer indicates that this difference is as much as 15% in some joints. How should this finding be documented?
Abnormal.

Which action should the registered nurse (RN) implement to complete an assessment for a client while using an interpreter?
Maintain eye contact with the client while listening to the translation.

A client is in the clinic for a routine health examination. The nurse notices the client appears underweight. Which question is most important for the nurse to ask when completing the health history of this client?
Have you experienced sudden weight loss?

A male executive is seen in the primary care clinic for a physical examination. While obtaining the client’s health history, the nurse inquires about his drug and alcohol use. The executive denies drug use, but reports that he has “two glasses of wine” per night. Which response is best for the nurse to provide?
“What effect do you think your use of alcohol may have on you?”

Which part of the body should the nurse examine when assessing for peripheral edema in a client with heart failure?
Ankles.

A client reports feeling increasingly fatigued for several months, and the nurse observes that the client’s lips are pale. Which additional data should the nurse collect based on this presentation?
Use of vitamin and iron supplements.

What is the best place for the nurse to hear lower lobe lung sounds with a stethoscope?
Posterior chest below the 3rd intercostalspace.

A registered nurse (RN) is performing a mini-mental state examination (MMSE) for a client who is being admitted to an assisted living community. Which communication techniques should the RN implement to decrease anxiety in the client? (Select all that apply.)
Use simple sentences during the examination.
Reduce environmental detractors during the examination.
Ask questions one at a time to decrease confusion.

The nurse is interviewing a client who reports having a persistent, productive cough during the winter caused by bronchitis. Which additional finding should the nurse assess for bronchitis?
Phlegm production and wheezing.

The nurse is assessing the posterior pharynx during a physical examination. Which technique should the nurse use?
Press the tongue down one side at a time with a tongue depressor.

The nurse is assessing a client who has a history of mitral stenosis. How should the nurse assess this client with a stethoscope to listen for this condition?
Place the bell on the 5th intercostal space, left midclavicular line.

Which statement is accurate about assessing the spleen?
It must be enlarged at least three times normal size for it to be palpable.

During an external examination of the eyes, the nurse gently palpates the eyes while the client’s eyelids are closed. The eyes are both very firm and resist movement back into the orbit. How should the nurse document this finding?
Abnormal finding.

Which tool should the nurse use when assessing the neurological status of a client with traumatic brain injury?
Glasgow Coma Scale.

The nurse is assessing a client with liver disease who is jaundice and exhibits scleral edema. During the health assessment, the nurse should implement which technique to determine evidence of hepatomegaly?
Use a bouncing motion to tap the middle finger placed within boundaries of the liver.

What is the best nursing response to an older client who has not mentioned incontinence during a genitourinary assessment?
Ask the client specifically about any leakage of urine.

The registered nurse (RN) is caring for an Asian client who refuses to make eye contact during conversations. How should the RN assess this client’s response?
The client is treating the nurse with respect.

The nurse is assessing a client for a hip flexion contracture. Which finding indicates a negative Thomas test when the client’s right knee is brought toward the chest?
The left leg remains on the table

*The Thomas test is performed by having the client bring one knee toward the chest while the other leg remains extended on the table. A positive Thomas test is elicited when the extended leg rises off the table when the opposite leg’s knee is brought up to the client’s chest, indicating hip flexor contracture. If the extended leg (the left leg, in this example) remains on the table, the test is negative.

The nurse is assessing a client who has a history of aortic regurgitation. Where should the nurse place the stethoscope diaphragm to listen for this condition?
2nd intercostal space along the right sternal border.

The nurse is assessing a client who has experienced a sudden onset of hearing loss in the right ear. Which finding should alert the nurse to a potentially serious medical condition that requires further evaluation?
There is no sign of associated infection.

Which information should the nurse obtain to identify the client’s self-perception of health status?
Health history

During the initial assessment, the nurse notes that a client has blurred vision with cloudy lenses. Which condition should the nurse document?
Cataracts.

Which condition is indicated by a fluorescent, yellow-green color when the nurse uses a Wood’s lamp toexamine a client’s skin lesions?
Fungal infection.

A client with dark skin is reporting a painful and itching area on the lower left leg. What should the nurse look for when assessing this client’s skin for inflammation?
Change in consistency.

A client reports pain when taking a deep breath. Which lung auscultation sound should the nurse anticipate hearing?
Pleural friction rub

A nurse is completing a nutritional assessment with a client. What is the easiest method for the nurse to use to get information about the client’s nutritional intake?
24-hour dietary recall

The nurse palpates a weak pedal pulse in the client’s right foot. Which assessment findings should the RN document that are consistent with diminished peripheral circulation? (Select all that apply.)
Diminished hair on legs.
Skin cool to touch.

The nurse is completing a physical assessment of a client who feel from a tree. The client’s abdomen is soft with hyperactive bowel sounds in all four quadrants. Which assessment technique should the nurse implement when evaluating the client’s spleen?
Percuss the splenic area as the client takes a deep breath.

The nurse enters an examination room to conduct a routine health assessment on an adolescent female client, who is accompanied by her mother. Which action by the nurse is likely to facilitate accurate responses to personal and social history questions?
Request that the mother leave the exam room.

While performing a mental status exam (MSE), the nurse asks a client to remember three unrelated words and repeat them later. The client was able to repeat the words as directed. Which computer documentation is accurate?
“Short-term memory is intact.”

Which technique should the nurse implement when performing a Weber test?
Place a vibrating tuning fork midline on top of the head

Which technique should the nurse use to assess a client for scoliosis?
Observe spine while the client is erect and bent forward

Which term should the nurse use to document in the client’s medical record for a high-pitched scratchy sound during auscultation of the heart?
Friction rub

While performing a head-to-toe assessment, the nurse assesses the client’s pupillary accommodation. During the second portion of the test, the nurse notes that the client’s pupils constrict and there is convergence of the axes of the eyes. What action should the nurse implement next?
Document a normal finding.

The nurse performs the Weber and Rinne tests to assess which cranial nerve?
VIII – vestibulocochlear

The nurse uses a tongue depressor to assess a client’s mouth. Which structure should the nurse be able to visualize?
Pharynx

As a part of a routine health assessment, the nurse assesses the kidneys as part of the abdominal assessment. Which assessment finding should the nurse conclude is normal when palpating the client’s right kidney?
A round smooth mass that slides between the fingers.

A client reports lower abdominal pain and a feeling of pressure in the bladder. Which assessment finding indicates acute urinary retention?
Dull sound percussed over bladder.

*Clients with acute urinary retention may present with lower abdominal pain and bladder distension. Percussion (tapping on the body wall) is performed to detect differences in pitch. A dull sound produced when percussing a distended urinary bladder is an indication of urinary retention.

The nurse examines the skin of an older adult client. Which skin variation is considered a normal finding for a client in this age group?
Lentigines.

*Lentigines or commonly referred to as liver spots are irregularly shaped dark spots on the skin caused by aging and extensive sun exposure. This skin variation is a normal finding in an older adult client.

During a client’s routine well-woman physical exam, the nurse examines the breasts. Which assessment technique should the nurse implement to evaluate for any abnormal lumps?
With both arms at client’s side, lift one arm and palpate the axilla.

The nurse is completing a physical exam on an adult client. Which thyroid finding is considered normal?
Gland is not palpable.

How should the nurse assess for lower extremity edema in a client who has been diagnosed with heart failure?
Measure bilateral ankle circumference with a non-stretchable tape measure.

A client has come to the clinic for a routine health assessment. What is the best assessment question for the nurse to ask a client after observing tophi on the client’s ear cartilage?
Have you had sudden and severe pain in the toes or feet?

During the interview portio of the health assessment, a nurse notes the person’s posture, physical appearance, and ability to converse. How should the nurse document these findings?
Objective.

The nurse is assessing a client who reports having shoulder pain. Which sign is the best indicator of a rotator cuff tear?
Inability to slowly lower the arm when abducted.

During cardiac auscultation, the nurse hears a split in the second heart sound when listening at the second left intercostal space of a male client. To assess this sound more fully, what action should the nurse implement?
Listen to the sound while observing the client’s respirations.

An older client has just returned to the room following a surgical procedure. Which pain scale should the nurse use when assessing the client’s pain level?
Verbal descriptor scale.

The nurse observes peristaltic movement in the left lower quadrant of a client’s abdomen. Which further assessment of the area should the nurse perform?
Observe the direction of movement.

The nurse is assessing a client’s middle lung lobe. What is the best location for the nurse to place a stethoscope diaphragm to hear normal lung sounds in this lobe?
4th intercostal space, right midclavicular line.

A Muslim male client refuses to let the female registered nurse (RN) listen to his breath sounds during the examination. How should the RN respond?
Request a male nurse or healthcare provider to perform the exam.

HESI Health Assessment Questions and Answers 1) The nurse hears bilateral louder, longer, and lower tones when percussing over the lungs of a 4-year old child. What should the nurse do next? a) Palpate over the area for increased pain and tenderness. b) Ask the child to take shallow breaths and percuss over the area again. c) Refer the child immediately because of an increased amount of air in the lungs. d) Consider this a normal finding for a child this age and proceed with the examination. 2) A patient has suddenly developed shortness of breath and appears to be in significant respiratory distress. After putting a call in to the physician and placing the patient on oxygen, which of these is the best action for the nurse to take when assessing the patient further? a) Count the patient’s respirations. b) Percuss the thorax bilaterally, noting any differences in percussion tones. c) Call for a chest x-ray and wait for the results before beginning an assessment. d) Inspect the thorax for any new masses and bleeding associated with respirations. 3) The nurse is teaching a class on basic assessment skills. Which of these statements is true regarding the stethoscope and its use? a) The slope of the earpieces should point posteriorly (toward the occiput). b) The stethoscope does not magnify sound but does block out extraneous room noise. c) The fit and quality of the stethoscope are not as important as its ability to magnify sound. d) The ideal tubing length should be 22 inches to dampen distortion of sound. 4) The nurse is preparing to use a stethoscope for auscultation. Which statement is true regarding the diaphragm of the stethoscope?
a) The diaphragm is used to listen for high-pitched sounds. b) The diaphragm is used to listen for low-pitched sounds. c) The diaphragm should be held lightly against the person’s skin to block out low-pitched sounds. d) The diaphragm should be held lightly against the person’s skin to listen for extra heart sounds and murmurs. 5) Before auscultating the abdomen for the presence of bowel sounds on a patient, the nurse should: a) Warm the end piece of the stethoscope by placing it in warm water b) Leave the gown on so that the patient does not get chilled during the examination c) Make sure that the bell side of the stethoscope is turned to the ―on‖ position d) Check the temperature of the room and offer blankets to the patient if he or she feels cold 6) The nurse will use which technique of assessment to determine the presence of crepitus, swelling, and pulsations? a) Palpation b) Inspection c) Percussion d) Auscultation
7) The nurse is preparing to use an otoscope for an examination. Which statement is true regarding the otoscope? a) The otoscope is often used to direct light onto the sinuses. b) The otoscope uses a short, broad speculum to help visualize the ear. c) The otoscope is used to examine the structures of the internal ear. d) The otoscope directs light into the ear canal and onto the tympanic membrane. 8) An examiner is using an ophthalmoscope to examine a patient’s eyes. The patient has astigmatism and is nearsighted. The use of which of these techniques would indicate that the examination is being performed correctly? a) Using the large full circle of light when assessing pupils that are not dilated b) Rotating the lens selector dial to the black numbers to compensate for astigmatism c) Using the grid on the lens aperture dial to visualize the external structures of the eye d) Rotating the lens selector dial to bring the object into focus 9) The nurse is unable to palpate the right radial pulse on a patient. The best action would be to: a) Auscultate over the area with a fetoscope b) Use a goniometer to measure the pulsations c) Use a Doppler device to check for pulsations over the area d) Check for the presence of pulsations with a stethoscope 10) The nurse is preparing to perform a physical assessment. The correct action by the nurse is reflected by which statement? a) The nurse performs the examination from the left side of the bed.
b) The nurse examines tender or painful areas first to help relieve the patient’s anxiety. c) The nurse follows the same examination sequence regardless of the patient’s age or condition. d) The nurse organizes the assessment so that the patient does not change positions too often. 11) A man is at the clinic for a physical examination. He states that he is ―very anxious‖ about the physical examination. What steps can the nurse take to make him more comfortable? a) Appear unhurried and confident when examining him. b) Stay in the room when he undresses in case he needs assistance. c) Ask him to change into an examining gown and take off his undergarments. d) Defer measuring vital signs until the end of the examination, which allows him time to become comfortable. 12) When performing a physical examination, safety must be considered to protect the examiner and the patient against the spread of infection. Which of these statements describes the most appropriate action the nurse should take when performing a physical examination? a) There is no need to wash one’s hands after removing gloves, as long as the gloves are still intact. b) Wash hands before and after every physical patient encounter. c) Wash hands between the examination of each body system to prevent the spread of bacteria from one part of the body to another.
d) Wear gloves throughout the entire examination to demonstrate to the patient concern regarding the spread of infectious diseases. 13) The nurse is examining a patient’s lower leg and notices a draining ulceration. Which of these actions is most appropriate in this situation? a) Wash hands and contact the physician. b) Continue to examine the ulceration and then wash hands. c) Wash hands, put on gloves, and continue with the examination of the ulceration. d) Wash hands, proceed with rest of the physical examination, and then continue with the examination of the leg ulceration. 14) During the examination, it is often appropriate to offer some brief teaching about the patient’s body or the examiner’s findings. Which of these statements by the nurse is most appropriate? a) ―Your atrial dysrhythmias are under control.‖ b) ―You have pitting edema and mild varicosities.‖ c) “Your pulse is 80 beats per minute. This is within the normal range.” d) ―I’m using my stethoscope to listen for any crackles, wheezes, or rubs.‖ 15) The nurse keeps in mind that the most important reason to share information and offer brief teaching while performing the physical examination is to help: a) The examiner feel more comfortable and gain control of the situation b) Build rapport and increase the patient’s confidence in the examiner c) The patient understand his or her disease process and treatment modalities d) The patient identify questions about his or her disease and potential areas of patient education
16) The nurse is examining an infant and prepares to elicit the Moro reflex at which time during the examination? a) When the infant is sleeping b) At the end of the examination c) Before auscultation of the thorax d) Halfway through the examination 17) When preparing to perform a physical examination on an infant, the nurse should: a) Have the parent remove all clothing except the diaper on a boy b) Instruct the parent to feed the infant immediately before the examination c) Encourage the infant to suck on a pacifier during the abdominal examination d) Ask the parent to briefly leave the room when assessing the infant’s vital signs 18) A 6-month-old infant has been brought to the well-child clinic for a check-up. She is currently sleeping. What should the nurse do first when beginning the examination? a) Auscultate the lungs and heart while the infant is still sleeping. b) Examine the infant’s hips because this procedure is uncomfortable. c) Begin with the assessment of the eye and continue with the remainder of the examination in a head-to-toe approach.

Document continues below
Discover more from:
Health Assess Interv & Eval (HSC 489)(HSC 489)
5 documents
Go to course
36
Hesi health assessment questions and answers
Hesi health assessment questions and answers
Health Assess Interv & Eval
89% (37)
33
Hesi health assessment combined
Hesi health assessment combined
Health Assess Interv & Eval
100% (5)
66
Hesi health assessment topics review latest update
Hesi health assessment topics review latest update
Health Assess Interv & Eval
100% (1)
3
Shadow Health Health History Documentation
Shadow Health Health History Documentation
Health Assess Interv & Eval
None
23
Hesi health assessment study guide
Hesi health assessment study guide
Health Assess Interv & Eval
None
10
Study Guide #3 Exam Nur 1212
Study Guide #3 Exam Nur 1212
Health Alterations Across the Lifespan I 5
100% (5)
d) Wake the infant before beginning any portion of the examination to obtain the most accurate assessment of body systems. 19) A 2-year-old child has been brought to the clinic for a well-child check-up. The best way for the nurse to begin the assessment is reflected by which statement? a) Ask the parent to place the child on the examining table. b) Have the parent remove all of the child’s clothing before the examination. c) Allow the child to keep a security object such as a toy or blanket during the examination. d) Initially focus interactions on the child, essentially ―ignoring‖ the parent, until the child’s trust has been obtained. 20) The nurse is examining a 2-year-old child and asks, ―May I listen to your heart now?‖ Which critique of the nurse’s technique is most accurate? a) Asking questions enhances the child’s autonomy. b) Asking the child for permission helps to develop a sense of trust. c) This is an appropriate statement because children at this age like to have choices. d) Children at this age like to say “No.” The examiner should not offer a choice when there is none 21) A woman brings her husband to the clinic for an examination. She is particularly worried because after a recent fall, he seems to have lost a great deal of his memory of recent events. Which statement reflects the nurse’s best course of action? a) The nurse should plan to perform a complete mental status examination. b) The nurse should refer him to a psychometrician. c) The nurse should plan to integrate the mental status examination into the history and physical examination.
d) The nurse should reassure his wife that memory loss after a physical shock is normal and will subside soon. 22) The nurse is conducting a patient interview. Which statement made by the patient should the nurse explore more fully during the interview? The patient states that he: a) ―Sleeps like a baby‖ b) Has no health problems c) ―Never did too good in school‖ d) Currently is not taking any medication 23) A patient is admitted to the unit after an automobile accident. The nurse begins the mental status examination and finds that the patient’s speech is dysarthric and that she is lethargic. The nurse’s best approach regarding this examination is to: a) Plan to defer the rest of the mental status examination b) Skip the language portion of the examination and go on to assess mood and affect c) Do an in-depth speech evaluation and defer the mental status examination to another time d) Go ahead and assess for suicidal thoughts because dysarthria is often accompanied by severe depression
24) A 19-year-old woman comes to the clinic at the insistence of her brother. She is wearing black combat boots and a black lace nightgown over the top of her other clothes. Her hair is dyed pink with black streaks throughout. She has several pierced holes in her nares and ears and is wearing an earring through her eyebrow and heavy black makeup. The nurse concludes: a) She probably doesn’t have any problems at all. b) She is just trying to shock people and her dress should be ignored. c) She has manic syndrome because of her abnormal dress and grooming. d) That more information should be gathered to decide whether her dress is appropriate. 25) A patient has been in the intensive care unit for 10 days. He has just been moved to the medicalsurgical unit, and the admitting nurse is planning to perform a mental status examination on him. During the tests of cognitive function the nurse would expect that he: a) May display some disruption in thought content b) Will state, ―I am so relieved to be out of intensive care‖ c) Will be oriented to place and person but may not be certain of the date d) May show evidence of some clouding of his level of consciousness 26) During a mental status examination, the nurse wants to assess a patient’s affect. The nurse should ask the patient which question? a) “How do you feel today?” b) ―Would you please repeat the following words?‖ c) ―Have these medications had any effect on your pain?‖ d) ―Has this pain affected your ability to get dressed by yourself?‖
27) The nurse is planning to assess new memory with a patient. The best way for the nurse to do this would be to: a) Administer the FACT test. b) Ask him to describe his first job. c) Give him the Four Unrelated Words test. d) Ask him to describe what television show he was watching before coming to the clinic. 28) A 29-year-old woman tells the nurse that she has ―excruciating pain‖ in her back. Which would be an appropriate response by the nurse to the woman’s statement? a) ―How does your family react to your pain?‖ b) ―That must be terrible. You probably pinched a nerve.‖ c) ―I’ve had back pain myself, and it can be excruciating.‖ d) “How would you say the pain affects your ability to do your daily activities?” 29) In recording the childhood illnesses of a patient who denies having had any, which note by the nurse would be most accurate? a) Patient denies usual childhood illnesses. b) Patient states he was a ―very healthy‖ child. c) Patient states sister had measles, but he didn’t. d) Patient denies measles, mumps, rubella, chickenpox, pertussis, and strep throat.
30) A patient tells the nurse that he is allergic to penicillin. What would be the nurse’s best response tothis information? a) ―Are you allergic to any other drugs?‖ b) ―How often have you received penicillin?‖ c) ―I’ll write your allergy on your chart so you won’t receive any penicillin.‖ d) “Please describe what happens to you when you take penicillin.” 31) The nurse is taking a family history. Important diseases or problems to ask the patient about specifically include: a) Emphysema b) Head trauma c) Mental illness d) Fractured bones 32) The review of systems provides the nurse with: a) Physical findings related to each system b) Information regarding health promotion practices c) An opportunity to teach the patient medical terms d) Information necessary for the nurse to diagnose the patient’s medical problem 33) Which of these statements represents subjective data the nurse obtained from the patient regarding the patient’s skin? a) Skin appears dry. b) No obvious lesions. c) Denies color change. d) Lesion noted lateral aspect right arm.
34) In an interview, the nurse may find it necessary to take notes to aid his or her memory later. Which statement is true regarding note-taking? a) Note-taking may impede the nurse’s observation of the patient’s nonverbal behaviors. b) Note-taking allows the patient to continue at his or her own pace as the nurse records what is said. c) Note-taking allows the nurse to shift attention away from the patient, resulting in an increased comfort level. d) Note-taking allows the nurse to break eye contact with the patient, which may increase his or her level of comfort. 35) The nurse asks, ―I would like to ask you some questions about your health and your usual daily activities so that we can better plan your stay here.‖ This question is found at the phase of the interview process. a) Summary b) Closing c) Body d) Opening or introduction
36) A woman has just entered the emergency department after being battered by her husband. The nurse needs to get some information from her to begin treatment. What is the best choice for an opening with this patient? a) ―Hello, Nancy, my name is Mrs. C.‖ b) ―Hello, Mrs. H., my name is Mrs. C. It sure is cold today!‖ c) ―Mrs. H., my name is Mrs. C. How are you?‖ d) “Mrs. H., my name is Mrs. C. I’ll need to ask you a few questions about what happened.” 37) During an interview, the nurse states, ―You mentioned shortness of breath. Tell me more about that.‖ Which verbal skill is used with this statement? a) Reflection b) Facilitation c) Direct question d) Open-ended question 38) A patient has finished giving the nurse information about the reason he is seeking care. When reviewing the data, the nurse finds that some information about past hospitalizations is missing. At this point, which statement by the nurse would be most appropriate to gather these data? a) ―Mr. Y., at your age, surely you have been hospitalized before!‖ b) ―Mr. Y., I just need permission to get your medical records from County Medical.‖ c) “Mr. Y., you mentioned that you have been hospitalized on several occasions. Would you tell me more about that?” d) ―Mr. Y., I just need to get some additional information about your past hospitalizations. When was the last time you were admitted for chest pain?‖
39) An Asian-American woman is experiencing diarrhea, which is felt to be ―cold‖ or ―yin.‖ The nurse expects that the woman is likely to try to treat it with: a) Foods that are “hot” or “yang” b) Readings and Eastern medicine meditations c) High doses of medicines thought to be ―cold‖ d) No treatment at all because diarrhea is an expected part of life 40) Illness is seen as a part of life’s rhythmic course and as an outward sign of disharmony within. This statement most accurately reflects the views about illness from the theory. a) Naturalistic b) Biomedical c) Reductionist d) Magicoreligious 41) An imaginary line connecting the highest point on each iliac crest would cross the vertebra. a) First sacral b) Fourth lumbar c) Seventh cervical d) Twelfth thoracic
42) The nurse is explaining to a patient that there are ―shock absorbers‖ in his back to cushion the spine and to help it move. The nurse is referring to his: a) Vertebral column b) Nucleus pulposus c) Vertebral foramen d) Intervertebral disks 43) During an interview the patient states, ―I can feel this bump on the top of both of my shoulders—it doesn’t hurt but I am curious about what it might be.‖ The nurse should tell the patient: a) ―That is your subacromial bursa.‖ b) “That is your acromion process.” c) ―That is your glenohumeral joint.‖ d) ―That is the greater tubercle of your humerus.‖ 44) The nurse is checking the range of motion in a patient’s knee and knows that the knee is capable of which movement(s)? a) Flexion and extension b) Supination and pronation c) Circumduction d) Inversion and eversion 45) A patient is visiting the clinic for an evaluation of a swollen, painful knuckle. The nurse notices that the knuckle above his ring on the left hand is swollen and that he is unable to remove his wedding ring. This joint is called the joint. a) Interphalangeal
b) Tarsometatarsal c) Metacarpophalangeal d) Tibiotalar 46) The nurse is percussing the seventh right intercostal space at the midclavicular line over the liver. Which sound should the nurse expect to hear? a) Dullness b) Tympany c) Resonance d) Hyperresonance 47) Which structure is located in the left lower quadrant of the abdomen? a) Liver b) Duodenum c) Gallbladder d) Sigmoid colon
48) A patient is having difficulty in swallowing medications and food. The nurse would document that this patient has: a) Aphasia b) Dysphasia c) Dysphagia d) Anorexia 49) The nurse suspects that a patient has a distended bladder. How should the nurse assess for this condition? a) Percuss and palpate in the lumbar region. b) Inspect and palpate in the epigastric region. c) Auscultate and percuss in the inguinal region. d) Percuss and palpate the midline area above the suprapubic bone. 50) The nurse is aware that one change that may occur in the gastrointestinal system of an aging adult is: a) Increased salivation b) Increased liver size c) Increased esophageal emptying d) Decreased gastric acid secretion 51) In assessing a 70-year-old man, the nurse finds the following: blood pressure 140/100 mm Hg; heart rate 104 and slightly irregular; split S2. Which of these findings can be explained by expected hemodynamic changes related to age? a) Increase in resting heart rate b) Increase in systolic blood pressure
c) Decrease in diastolic blood pressure d) Increase in diastolic blood pressure 52) A 45-year-old man is in the clinic for a routine physical. During the history the patient states that he’s been having difficulty sleeping. ―I’ll be sleeping great and then I wake up and feel like I can’t get my breath.‖ The nurse’s best response to this would be: a) ―When was your last electrocardiogram?‖ b) ―It’s probably because it’s been so hot at night.‖ c) “Do you have any history of problems with your heart?” d) ―Have you had a recent sinus infection or upper respiratory infection?‖ 53) In assessing a patient’s major risk factors for heart disease, which would the nurse want to include when taking a history? a) Family history, hypertension, stress, age b) Personality type, high cholesterol, diabetes, smoking c) Smoking, hypertension, obesity, diabetes, high cholesterol d) Alcohol consumption, obesity, diabetes, stress, high cholesterol
54) The mother of a 3-month-old infant states that her baby has not been gaining weight. With further questioning, the nurse finds that the infant falls asleep after nursing and wakes up after a short amount of time, hungry again. What other information would the nurse want to have? a) The infant’s sleeping position b) Sibling history of eating disorders c) Amount of background noise when eating d) Presence of dyspnea or diaphoresis when sucking 55) In assessing the carotid arteries of an older patient with cardiovascular disease, the nurse would: a) Palpate the artery in the upper one third of the neck b) Listen with the bell of the stethoscope to assess for bruits c) Palpate both arteries simultaneously to compare amplitude d) Instruct patient to take slow deep breaths during auscultation 56) The nurse is reviewing the technique of palpating for tactile fremitus with a new graduate. Which statement by the graduate nurse reflects a correct understanding of tactile fremitus? a) ―Tactile fremitus is caused by moisture in the alveoli.‖ b) ―Tactile fremitus indicates that there is air in the subcutaneous tissues.‖ c) “Tactile fremitus is caused by sounds generated from the larynx.” d) ―Tactile fremitus reflects the blood flow through the pulmonary arteries.‖ 57) The nurse is observing the auscultation technique of another nurse. The correct method to use when progressing from one auscultatory site on the thorax to another is comparison. a) Side-to-side b) Top-to-bottom
c) Posterior-to-anterior d) Interspace-by-interspace 58) When auscultating the lungs of an adult patient, the nurse notes that over the posterior lower lobes low-pitched, soft breath sounds are heard, with inspiration being longer than expiration. The nurse interprets that these are: a) Sounds normally auscultated over the trachea b) Bronchial breath sounds and are normal in that location c) Vesicular breath sounds and are normal in that location d) Bronchovesicular breath sounds and are normal in that location 59) The nurse is auscultating the chest in an adult. Which technique is correct? a) Instruct the patient to take deep, rapid breaths. b) Instruct the patient to breathe in and out through his or her nose. c) Use the diaphragm of the stethoscope held firmly against the chest. d) Use the bell of the stethoscope held lightly against the chest to avoid friction.
60) The nurse is percussing over the lungs of a patient with pneumonia. The nurse knows that percussion over an area of atelectasis in the lungs would reveal: a) Dullness b) Tympany c) Resonance d) Hyperresonance 61. An 85-year-old man has come in for a physical examination, and the nurse notices that he uses a cane. When documenting general appearance, the nurse should document this information under the section that covers: A) posture. B) mobility. C) mood and affect. D) physical deformity. 62. The nurse is performing a vision examination. Which of these charts is most widely used for vision examinations? A) Snellen B) Shetllen C) Smoollen D) Schwellon 63. After the health history has been obtained, and before beginning the physical examination, the nurse should ask the patient to first: A) empty the bladder. B) completely disrobe.
C) lie on the examination table. D) walk around the room. 64. During a complete health assessment, how would the nurse test the patient’s hearing? A) By observing how the patient participates in normal conversation B) Using the whispered voice test C) Using the Weber and Rinne tests D) Testing with an audiometer 65. A patient states, ―Whenever I open my mouth real wide, I feel this popping sensation in front of my ears.‖ To further examine this, the nurse would: A) place the stethoscope over the temporomandibular joint and listen for bruits. B) place the hands over his ears and ask him to open his mouth ―really wide.‖ C) place one hand on his forehead and the other on his jaw and ask him to try to open his mouth. D) place a finger on his temporomandibular joint and ask him to open and close his mouth. 66. The nurse has just completed an examination of a patient’s extraocular muscles. When documenting the findings, the nurse should document the assessment of which cranial nerves? A) II, III, VI B) II, IV, V C) III, IV, V
D) III, IV, VI 67. A patient’s uvula rises midline when she says ―ahh‖ and she has a positive gag reflex. The nurse has just tested which cranial nerves? A) IX, X B) IX, XII C) X, XII D) XI, XII 68. During an examination, the nurse notices that a patient is unable to stick out his tongue. Which cranial nerve is involved with successful performance of this action? A) I B) V C) XI D) XII 69. A patient is unable to shrug her shoulders against the nurse’s resistant hands. What cranial nerve is involved with successful shoulder shrugging? A) VII B) IX C) XI D) XII 70. During an examination, a patient has just successfully completed the finger-to-nose and the rapid-alternating-movements tests and is able to run each heel down the opposite shin. The nurse will conclude that the patient’s function is intact. A) occipital B) cerebral
C) temporal D) cerebellar 71. A 5-year old child is in the clinic for a checkup. The nurse would expect him to: A) have to be held on his mother’s lap. B) be able to sit on the examination table. C) be able to stand on the floor for the examination. D) be able to remain alone in the examination room. 72. When the nurse performs the confrontation test, the nurse has assessed: A) extraocular eye muscles (EOMs). B) pupils (PERRLA). C) near vision. D) visual fields. 73. Which of these statements is true regarding the complete physical assessment? A) The male genitalia should be examined in the supine position. B) The patient should be in the sitting position for examination of the head and neck. C) The vital signs, height, and weight should be obtained at the end of the examination.
D) To promote consistency between patients, the examiner should not vary the order of the assessment. 74. Which of these statements is true regarding the recording of data from the history and physical examination? A) Use long, descriptive sentences to document findings. B) Record the data as soon as possible after the interview and physical examination. C) If the information is not documented, then it can be assumed that it was done as a standard of care. D) The examiner should avoid taking any notes during the history and examination because of the possibility of decreasing rapport with the patient. 75. Which of these is included in assessment of general appearance? A) Height B) Weight C) Skin color
D) Vital signs 76. A male patient is admitted for observation after being hit on the head with a baseball bat. Six hours after admission the client attempts to crawl out of bed and asks the nurse why there are so many bugs in his bed. His vital signs are stable and the pulse oximeter reading is 98% on room air. Which intervention should the nurse perform first? A) Administer oxygen per nasal cannula at 2L/min B) Plan to check his vitals signs again in 30 mins C) Notify the healthcare provider of the change in mental status D) Ask the client why he thinks there are bugs in the bed. 77. A client with hemiparesis needs assistance transferring from the bed to the wheelchair. The nurse assists the clients to a sitting position on the side of the bed. Which action should the nurse implement next? A) Flex the hips and knee and align the knees with the client’s knees for safety B) Allow the client to sit on the side of the bed for a few minutes before transferring C) Place the client’s weight-bearing or strong leg forward and the weak foot back D) Grasp the transfer belt at the client’s sides to provide movement of the client. Case Studies Seizure Disorder
A 12-year-old boy began to complain of frequent headaches 4 months before his hospital admission. On the day of his admission, he had a major motor seizure, which his parents observed. During the seizure he lost bladder and bowel control. On physical examination he
appeared to be in deep postictal sleep. He had no focal neurologic signs. On examination of the optic fundi, no evidence of papilledema was found. 1. What are the major assessments that the nurse should make during seizure activity? 2. Why is the EEG a priority study for patients with seizure disorders? Anorexia Nervosa B.J., a 14-year-old white girl, was taken to her family nurse practitioner by her mother because of cessation of menses. Once an overweight child, Becky recently had a severe weight loss and was described by her mother as anxious, irritable, and depressed. Despite her weight of 85 pounds (and height of 64 inches), B.J. was jogging long distances daily. The physical examination was compatible with signs of starvation (such as sparse, dry hair; dry, flaky skin; muscle wasting; and red, swollen lips). Studies Results Triceps skinfold thickness (TSF) 65% standard Midarm circumference (MAC) 65% standard Midarm muscle circumference (MAMC) 65% standard Hemoglobin (Hgb) 10 g/dL (normal: 12 g/dL) Hematocrit (Hct) 31% (normal: 36%) Total iron-binding capacity (TIBC) 210 mcg/dL (normal: 250-420 mcg/dL) Serum albumin 2.8 g/dL (normal: 3.2-4.5 g/dL) Total protein 4 g/dL (normal: 6-8 g/dL) Total lymphocyte count 1200/mm3 (normal: 1500-3000/mm3) Blood urea nitrogen (BUN) 30 mg/dL (normal: 5-20 mg/dL)
24-Hour urine for creatinine Decreased when compared with expected creatinine clearance based on height and sex Serum triglycerides 200 mg/dL (normal: 40-150 mg/dL) Skin testing with common antigens Delayed sensitivity to mumps, purified protein derivative (PPD), and Candida Diagnostic Analysis The triceps skinfold thickness (TSF), which estimates the amount of subcutaneous fat, reflected the depleted caloric stores in the body. The midarm muscle circumference (MAMC), which is calculated using the midarm circumference (MAC) and the TSF, reflected moderate to severe muscle protein depletion as a result of catabolism. The decreased hemoglobin and hematocrit levels reflected anemia because of iron and folic acid deficiency. The TIBC level reflected transferrin concentration (p. 336), which is a sensitive and early indicator of protein deficiency. The decreased plasma albumin and protein levels correlated with protein depletion, fatty liver, and edema. The elevated BUN level was the result of catabolism. The decreased lymphocyte count is further evidence of protein malnutrition. Because the 24-hour urinary excretion of creatinine is approximately proportional to lean body mass, its decrease reflected a severe degree of muscle protein depletion and decreased muscle mass. The antigen skin test showed decreased immunocompetence, which is seen in nutritional starvation. This also reflects the impaired ability of the white blood cells to fight infection.
Based on the results of these tests and a detailed family history, B.J. was placed in the hospital in an adolescent unit for patients with anorexia. After several weeks of nutritional counseling and behavior modification, she was discharged to home. Individual and family counseling were continued over the next year. Question 1. What kind of questions were probably asked while obtaining the detailed family history? Thrombophlebitis N.S. was a 32-year-old nursing assistant who was admitted to the hospital complaining of a painful, swollen right leg. She was otherwise in good health. On physical examination, her right leg was one and a half times the size of her left leg. The right calf was tender, and 3+ pitting edema was present. Questions 1. If N.S. were to develop an acute episode of shortness of breath, what would you do? 2. What would be included in the diagnostic evaluation of this problem? Please Review the following topics What is Evidence Based Practice Assessment of the Adolescents: HEADSSS Why is it important to obtain a thorough drug history from a client? Evaluate patient knowledge about proper disposal of drugs: How do I dispose of unused prescription and OTC drugs?
1) Patient need to ask the Pharmacy that sold the drug to dispose of it 2) Ask if the community has a community household hazardous waste collection program ( Lacking this preferred option, placement in the garbage container with transport to a landfill is a practical solution) 3) Keep drug in original container with label and caps Basic components for Nursing Health History Interview Techniques Approach in Clinical Settings Deep Tendon Reflex- Location of tendons for eval of DTRs and grading Clinical manifestation of Dehydration adults and infants
Assessing the head and neck- what are your findings (glands, lymph nodes ect.) ( fontanelles and sutures in infant) Gathering Health History on a breast cancer patient Menstrual Cycle Hypothalamic-Pituitary cycle and ovarian cycle and endometrial cycle in relation to menstrual cycle What is climacteric phase? Changes in thyroid gland and thyroid hormone during pregnancy. What are the changes? Heart- Cardiac cycle and its events— Characteristics of heart sounds S1- S2 During health promotion regarding the heart-nurse should explain the risk factors for heart disease including high dietary intake of saturated fat or cholesterol, lack of regular aerobic exercise, smoking, excess weight, stressful lifestyle, hypertension, and family history of heart disease. Position of an infant’s heart Assessment of signs and symptoms suggestive of heart disease and what other risk factors may have a patient prone to heart disease ( gathering health history) Anatomical sites for assessment of cardiac function What is PMI Review auscultation of murmurs—which are low pitch sound (ie S3-S4) grading of murmurs Dysrhythmia Hypoxemia Congestive heart failure Pg 486 of your text –Cyanosis or pallor in cardiac pt P 468
Abdominal assessment- What are borborygmi sounds Assessing Bowel Sounds sites, Common findings and what do they mean? ie. Absence of bowel sounds in 5 mins = Peritonitis, paralytic ileus, hypokalemia Bruit= ? , Hum or friction rub= ? Assessing patients for allergies? Language is a major barrier to health care. Use of Interpreter during health history interview of a patient— Communicating with culturally diverse patients
Proper use of Stethoscopes during auscultation? Skin assessment – Drug allergies vs Allergic reaction from substances other than medication and assessing the following mild allergic reactions: Symptoms- Urticuria or hives, eczema or rash, pruritus, rhinitis, wheezing, angioedema or fever. ( know these terms) Know Hypothyroidism and goiter Page 274-276 of your book Nutritional deficiencies and assessment ( height , weight ect..) –The hair –primary function and importance in assessing the hair and nails, –Clubbing of fingers Assessing intimate partner violence Know Terms in assessing Patient Level of Consciousness: Awake and Alert, Lethargic or Somnolent, Obtunded, Stupor, Semicoma, Comatose Assessing Mood, affect and speech. Assessing abnormal breath sound pg 439-453 Assessment of Anus, Rectum, and Prostate: abnormalities such as Melena, Dyschezia and Hematochezia Page 712 (Hematochezia is bright red blood in the stool due to lower GI bleed) Hemorrhoids- health promotion pg 720 Assessment of Pain— Gallbladder and Assessment or Referred Abdominal Pain Pg559 Congenital Heart Defects Pg 493 Table 19-9 Tetralogy of Fallot ( Note Clinical Data- S & O Let’s talk about this— A 3 month old infant known to have tetralogy of Fallot is seen in the ER because of a 2 day
history of diarrhea, low-grade fever, and poor oral intake. When blood test are obtained he becomes acutely cyanotic with rapid, shallow respirations, Treatment- Place infant in knee-chest position Employ calm, comforting approach Administer Oxygen 100% by Face mask Give Morphine Subcutaneously or through existing IV line Begin IV fluid replacement and volume expansion if needed Aggressive pulmonary hygiene
Chest physiotherapy Give antibiotics all are important Interventions

Leave a Comment

Scroll to Top